🔙 Quay lại trang tải sách pdf ebook Tạp Chí Epsilon Số 9 Ebooks Nhóm Zalo ÍCH GÌ, TOÁN HỌC? Hà Huy Khoái HÀM MOEBIUS VÀ ĐỊNH LÝ PHẦN DƯ TRUNG HOA Phùng Hồ Hải DẪN NHẬP VỀ HÀM ZETA RIEMANN VÀ PHÉP BIẾN ĐỔI MELLIN Ngô Bảo Châu GIẢI NOBEL CỦA EINSTEIN HAY LÀ SÓNG ĐIỆN THOẠI CÓ GÂY UNG THƯ HAY KHÔNG Đàm Thanh Sơn NO tháng 6 - 2016 09 VÀ CÁC CHUYÊN MỤC KHÁC Không có gì gần với thực tiễn hơn là một lí thuyết đẹp” ISAAC NEWTON “Tích của hai số nguyên, mỗi số là tổng hai bình phương, cũng là tổng của hai bình phương. DIOPHANTUS CHỦ BIÊN: Trần Nam Dũng BIÊN TẬP VIÊN: Võ Quốc Bá Cẩn Trần Quang Hùng Nguyễn Văn Huyện Nguyễn Tiến Lâm Lê Phúc Lữ Ngô Quang Dương Nguyễn Tất Thu Đặng Nguyễn Đức Tiến NO tháng 6 - 2016 09 LỜI NGỎ CHO EPSILON SỐ 9 Ban Biên tập Epsilon Epsilon số 9, ra mắt vào tháng 6, 2016 kỷ niệm chặng đường một năm rưỡi đầy nỗ lực của Ban Biên tập, các tác giả và cộng tác viên cũng như đầy ân tình từ phía độc giả đã ủng hộ chúng tôi suốt từ buổi ban đầu. Số tháng 9 đến được với các bạn cũng là thời điểm mùa hè bắt đầu rộn rã. Những kỳ nghỉ vẫy gọi và hoa phượng giăng đầy sân trường, lớp học, sắc đỏ lan khắp nắng, khắp những con đường khiến bất kì ai vô tình lướt ngang qua đều man mác biết bao hoài niệm. Không khí tháng sáu đã làm sống lại trong chúng tôi, những người thực hiện Epsilon, rất nhiều ký ức đẹp đẽ. Vì vậy chúng tôi muốn dành lần ra mắt này để giới thiệu về chủ đề Trung học Cơ sở. Bên cạnh những chuyên mục định kỳ, các đề tài và kiến thức toán trung học cơ sở sẽ lần lượt được trình bày trong số tạp chí mùa hè này. Ban biên tập và các cộng tác viên hy vọng rằng, Epsilon số 9 sẽ góp thêm niềm vui học tập vào mùa hè tràn ngập niềm vui của các em học sinh, cũng như làm sống lại ký ức một thuở bảng đen, mực tím với những độc giả đã qua tuổi hoa niên. Trân trọng. Ban Biên tập Epsilon. 3 MỤC LỤC Ban Biên tập Epsilon Lời ngỏ cho Epsilon số 9 . . . . . . . . . . . . . . . . . . . . . . . . . . . . . . . . . . 3 Hà Huy Khoái Ích gì, toán học? . . . . . . . . . . . . . . . . . . . . . . . . . . . . . . . . . . . . . . . 5 Phùng Hồ Hải Hàm Moebius và định lý phần dư Trung Hoa . . . . . . . . . . . . . . . . . . . . . . . . 13 Ngô Bảo Châu Dẫn nhập về hàm zeta Riemann và phép biến đổi Mellin . . . . . . . . . . . . . . . . . . 15 Đàm Thanh Sơn Giải Nobel của Einstein hay là sóng điện thoại có gây ung thư hay không . . . . . . . . . 21 Đặng Minh Tuấn Hệ mật mã khóa công khai dựa trên đường cong Elliptic - Tổng quan về hệ mật mã khóa công khai . . . . . . . . . . . . . . . . . . . . . . . . . . . . . . . . . . . . . . . . . . 23 Đặng Nguyễn Đức Tiến Nghịch lý trai gái . . . . . . . . . . . . . . . . . . . . . . . . . . . . . . . . . . . . . . 43 Job Bouwman Phép biến đổi Fourier có ý nghĩa vật lý gì? . . . . . . . . . . . . . . . . . . . . . . . . . 47 Kiều Đình Minh Tập hợp trù mật và ứng dụng . . . . . . . . . . . . . . . . . . . . . . . . . . . . . . . . 53 Đinh Trung Hòa About means of non-negative numbers and positive definite matrices . . . . . . . . . . . 67 Trịnh Huy Vũ Điểm Kosnita và một số đường thẳng đi qua nó . . . . . . . . . . . . . . . . . . . . . . 73 4 Tạp chí Epsilon, Số 09, 06/2016 Trần Quang Hùng, Dương Ánh Ngọc Định lý Sawayama và Thébault trong các bài toán hình học thi Olympic . . . . . . . . . 91 Lê Phúc Lữ Về bài toán tam giác 80-80-20 . . . . . . . . . . . . . . . . . . . . . . . . . . . . . . . 115 Nguyễn Tài Chung Sử dụng tổng tích phân để tính giới hạn dãy số . . . . . . . . . . . . . . . . . . . . . . . 121 Nguyễn Ngọc Giang Sáng tạo với một bài toán hình học Trung học cơ sở . . . . . . . . . . . . . . . . . . . . 133 Trung Dũng “Nếu bạn không nuôi dưỡng, đam mê sẽ từ bỏ bạn" - Trò chuyện về con đường đến với CMU của Phạm Hy Hiếu . . . . . . . . . . . . . . . . . . . . . . . . . . . . . . . . . . 149 Võ Quốc Bá Cẩn Về phong trào Olympic Toán ở Saudi Arabia . . . . . . . . . . . . . . . . . . . . . . . . 159 Trần Nam Dũng Bài toán hay lời giải đẹp . . . . . . . . . . . . . . . . . . . . . . . . . . . . . . . . . . 163 Trần Nam Dũng Đồng nhất thức Brahmagupta-Fibonacci và ứng dụng . . . . . . . . . . . . . . . . . . . 165 Ban biên tập Một số bài toán trong đề thi vào các trường chuyên . . . . . . . . . . . . . . . . . . . . 173 Ban biên tập Tuyển chọn các đề thi Olympic năm 2016 dành cho học sinh THCS . . . . . . . . . . . . 183 Ban biên tập Lời giải đề thi Toán quốc tế Formula of Unity - The Third Millennium . . . . . . . . . . 189 Trần Nam Dũng Các vấn đề cổ điển và hiện đại . . . . . . . . . . . . . . . . . . . . . . . . . . . . . . . 197 5 Tạp chí Epsilon, Số 09, 06/2016 6 ÍCH GÌ, TOÁN HỌC? (Hà Huy Khoái) Bài giảng đại chúng, kỷ niệm 5 năm Chương trình quốc gia phát triển Toán học và thành lập Viện nghiên cứu cao cấp Toán (VIASM), 20/12/2015. 1. Câu hỏi “Ích gì?” Trong những dịp kỷ niệm 5 năm, 10 năm, ... của tổ chức nào đó, người ta thường liệt kê những việc đã làm, những kết quả đạt được. Thực chất là cố gắng “chứng minh” rằng, việc thành lập cái tổ chức đó là cần thiết, rằng nó có ích. Vậy nên câu hỏi “Ích gì, Chương trình quốc gia phát triển Toán học?”, “Ích gì, VIASM?”, nếu không được phát biểu một cách công khai, thì chắc chắn cũng lởn vởn trong đầu không ít người, như nó đã từng được đặt lên bàn của những nhà hoạch định chính sách, của Bộ Tài chính, Bộ Giáo dục và Đào tạo. Vậy nên, khi được đề nghị làm một “bài giảng đại chúng”, tôi đã chọn đề tài “Ích gì, Toán học?”. Mà người “mách nước” cho tôi chọn đề tài đó lại không là một nhà toán học, mà là ... Chế Lan Viên! Hình như ông cũng đã từng trăn trở với câu hỏi “Ích gì Thơ ca? Ích gì, Nghệ thuật?”. Ích gì? Chế Lan Viên – Di cảo Khéo rồi mất giống bò sữa, hoạ mi, ngựa đua, gà chọi ... Khó gì? Ta không giữ, không nuôi thì nó mất Giống các nhà thơ cũng vậy Tuyết trên non cao không ai thấy, Giống nàng tiên, ông Bụt hiện trong mơ Mà chả cần ai giết Chỉ thôi yêu là nó chết Chỉ cần bâng quơ, vu vơ đặt ra câu hỏi Trịnh trọng cái bâng quơ, vu vơ ấy Hỏi rằng: Ích gì họa mi? Ích gì bò sữa? Ích gì xạ hương? Ích gì thi sĩ? Ích gì cái hôn? Ích gì giấc mơ? Ông Bụt ích gì? ... Đến nhà thơ cũng khó trả lời những câu hỏi như “Ích gì cái hôn? Ích gì thi sĩ? Ích gì giấc mơ? Ích gì ông Bụt?”, nói chi đến những người cầm túi tiền để cân nhắc đầu tư! Nhưng, nếu cứ luôn luôn đặt ra câu hỏi “Ích gì?”, lại còn “trịnh trọng” cái câu hỏi ấy, thì đến bò sữa còn chết, huống gì họa mi và giấc mơ? Với tôi, bài thơ trên còn thiếu một câu: Ích gì, Toán học? 7 Tạp chí Epsilon, Số 09, 06/2016 2. Đối tượng của toán học: Tìm về cội nguồn Lo lắng như Chế Lan Viên cũng phải, nhưng làm sao có thể lảng tránh câu hỏi “Ích gì”? Nhất là đối với Toán học, khi nhìn sang “bên cạnh”, hình như chưa có ai đặt ra câu hỏi: “Ích gì, Vật lý? Ích gì, Sinh học?”. Ích gì, Vật lý? Dễ trả lời thôi, vì vật lý học nghiên cứu vật chất và chuyển động của chúng trong không gian và thời gian. Có ai lại không cần những kiến thức đó? Ích gì, Sinh học? Dễ trả lời thôi, vì sinh học nghiên cứu các cơ thể sống và tương tác của chúng với môi trường. Có ai lại không cần những kiến thức đó? Nhưng “Ích gì, Toán học? Toán học nghiên cứu cái gì?” thì lại là câu hỏi không dễ trả lời. Để hiểu đối tượng của Toán học, phải tìm về cội nguồn của nó. Tức là phải tìm đến “Cơ sở” của Euclid. Trước khi cuốn “Cơ sở” ra đời (khoảng 300 năm trước Công nguyên), Toán học chưa phải là một khoa học độc lập. Nó “lẫn” vào Triết học và Thiên văn học. Bắt đầu với những “định nghĩa cơ bản” về những đối tượng của Toán học, trong Cơ sở - cuốn I, Euclid đưa ra 23 định nghĩa cơ bản. Xin nhắc lại ba trong số đó, định nghĩa thứ 1, 2 và 15 : α΄v. Σημεῖόν ἐστιν, οὗ μέρος οὐθέν. β΄v. Γραμμὴ δὲ μῆκος ἀπλατές. ιε΄v. Κύκλος ἐστὶ σχῆμα ἐπίπεδον ὑπὸ μιᾶς γραμμῆς περιεχόμενον [ἣ καλεῖται περιφέρεια], πρὸς ἣν ἀφ΄ ἑνὸς σημείου τῶν ἐντὸς τοῦ σχήματος κειμένων πᾶσαι αἱ προσπίπτουσαι εὐθεῖαι [πρὸς τὴν τοῦ κύκλου περιφέρειαν] ἴσαι ἀλλήλαις εἰσίν. Dịch nghĩa 1. Điểm là một cái không có kích thước. 2. Đường là cái chỉ có chiều dài, không có chiều rộng. 15. Đường tròn là một hình phẳng chỉ gồm một đường duy nhất (gọi là chu vi), (sao cho) mọi đường thẳng xuất phát (đến chu vi) từ một điểm nằm bên trong hình đều bằng nhau. Như vậy, Toán học nghiên cứu những sự vật ... không hề tồn tại trong thực tế! Không thể tìm ra một “vật thể” không có kích thước, cũng như không thể tìm ra một cái gì đó không có chiều rộng. Và hiển nhiên, cái đường tròn “lý tưởng” của Toán học không thể tồn tại, người ta chỉ nhìn thấy “vành nón tròn, mặt trời tròn”, hay “khuôn trăng đầy đặn” của Thuý Vân tròn như Mặt trăng! Vậy thì, ích gì, cái khoa học nghiên cứu những sự vật không hề tồn tại? Tìm về cội nguồn lại không cho ta câu trả lời, mà ngược lại, hình như còn làm ta bối rối thêm. Nhưng, phải chăng những gì không tồn tại trong thực tế đều vô ích, đều cần phải biến mất sau câu hỏi “Ích gì?” Ta thử tìm về Pablo Picasso, hoạ sĩ vĩ đại của thế kỷ XX. Người ta nhìn thấy Picaso không chỉ trong những bức tranh ông để lại, mà cả trong những vật dụng hàng ngày. Ông làm thay đổi quan niệm của chúng ta về cái đẹp. Và điều kỳ diệu đó Picasso làm được, khi đưa ta về tận cùng của bản chất sự vật. Một trong những đề tài thường gặp trong tranh Picasso là bò tót. Con bò tót, đấu bò gần như là biểu tượng của Tây Ban Nha. Nhưng trong tranh Picasso, bò tót tượng trưng cho sức mạnh u tối của chủ nghĩa phát xít Franco những năm 30 của thế kỷ trước. Ta hãy xem cách Picasso vẽ bò tót: 8 Tạp chí Epsilon, Số 09, 06/2016 Nhưng đó hiển nhiên chưa phải là “con bò của Picaso” vì nó hoàn toàn giống như con bò ta vẫn nhìn thấy trên đấu trường. Con bò nổi tiếng của Picaso “đơn giản”, và “xấu” hơn nhiều: Nhưng để đi đến con bò như bức vẽ của trẻ con đó, nhà hoạ sĩ vĩ đại đã phải trải qua một quá trình sáng tạo nhọc nhằn. Ta hãy xem cách ông đi từ con bò giống như thật đến con bò với nét vẽ trẻ con: 9 Tạp chí Epsilon, Số 09, 06/2016 Như vậy, Picaso đã đi từ “con bò tót” đến “khái niệm bò”! Con bò “khái niệm” của Picasso với bộ sừng đáng sợ, với bộ óc nhỏ như một “điểm” của Euclid, đã thể hiện đầy đủ cái sức mạnh ngu muội của chủ nghĩa phát xít những năm 30. Hơn hai ngàn năm trước, Euclid cũng bằng cách đó đi từ “mặt trời tròn, vành trăng tròn” đến cái “đường tròn” của toán học. Cái không có trong thế giới thực tại lại mô tả chân thực nhất thế giới thực tại, vì nó đưa ta về với bản chất. Phải chăng đó là lý do giải thích việc các lý thuyết toán học cho ta công cụ mô tả chính xác thế giới tự nhiên. Nói như Galilei, Thượng đế viết nên tự nhiên bằng ngôn ngữ của toán học. 3. Nghề làm Toán Nhiều người hỏi bác Tôm (René Thom, nhà toán học Pháp, giải thưởng Fields) về nghề làm Toán. Thấy khó nói quá, bác bèn kể chuyện săn rồng. Chuyện rằng, xưa bên Trung Quốc, có anh chàng học nghề đi săn. Anh chẳng chịu học săn hổ, săn lợn, mà lại học nghề săn Rồng! Nghề này khó lắm, phải thực tập nhiều. Bởi thế nên khi anh ta thạo nghề thì trên thế gian chẳng còn lấy một con Rồng nào! Có người hỏi: Bây giờ sống bằng nghề gì? Đáp: Đi dạy nghề săn Rồng! Bác Tôm nói: Làm Toán tức là đi dạy nghề săn Rồng vậy! (thảo nào chẳng có chú Rồng nào dám bén mảng đến nhà bác Tôm!). Thế thì, làng nước đâu có cần cái anh săn Rồng ấy. Có còn Rồng nữa đâu mà học nghề săn rồng? Ấy chết, đừng vội nói thế. Rồng thì chẳng còn, nhưng có khi vẫn phải học nghề săn Rồng đấy. Nếu anh đi học nghề săn lợn thì chắc gì đã bắn được hổ? Mà học nghề săn hổ thì chắc gì bắn được voi? Nhưng nếu đã thạo nghề săn Rồng thì hổ, báo, sư tử, voi, ... chắc chắn đều săn được tuốt! Này nhé, Rồng có thân như cá sấu, móng vuốt như hổ, đầu sư tử, ẩn hiện như trăn, vậy mà còn không thoát được tay anh săn Rồng, thì chẳng nói gì đến hổ, báo, voi, trăn, mà sau này có “nhân bản” ra con nào nữa, anh ta cũng chẳng sợ! Thành ra, đã định học nghề đi săn thì hãy cứ học nghề săn Rồng! Từ cá sấu, hổ, sư tử, trăn, ... người xưa “trừu tượng hóa” thành con Rồng. Cũng như thế, từ thực tiễn, người ta trừu tượng hóa thành Toán học. Câu chuyện đơn giản của bác Tôm mà thâu tóm được cả cái mạnh, và cái yếu, của Toán học là vậy. Khi đã trừu tượng hoá để tìm đến bản chất, Toán học không phải bao giờ cũng dễ dàng trở về với thực tại, vốn là nơi xuất phát của nó. Thậm chí, người ta còn nghi ngờ cái khả năng nó có thể quay về với thực tại. Bởi thế nên mới có người hỏi khích bác Tôm: “Mấy cái anh làm Toán gàn dở bịa ra những phương trình, vi phân, tích phân,. . . gì gì nữa nhỉ, thực tế làm gì có? Bọn họ chỉ ngồi chơi cái trò chơi trí tuệ đấy thôi”! Bác Tôm hỏi lại: “Này nhé, nếu anh đánh rơi cái nhẫn trong góc nhà kho bừa bộn, tối om, mà lại không có đèn, thì anh tìm nó ở đâu”? Anh chàng nọ ngạc nhiên: “Hỏi lạ nhỉ, thì chui vào đó mà tìm chứ ở đâu nữa”! Bác Tôm cười: “Thế thì có khi mấy tháng trời vẫn chưa tìm ra. Cứ như tôi thì tôi sẽ chạy ra dưới ngọn đèn sáng mà tìm vậy”! Anh chàng được mẻ cười vỡ bụng: “Mấy anh làm Toán gàn quá đi mất, biết tỏng tòng tong là nhẫn rơi trong góc nhà kho, mà lại ra dưới đèn tìm thì có mà suốt đời tìm cũng không thấy”. Ấy vậy mà cái anh đồ (Toán) gàn dở chẳng dại lắm đâu. Này nhé, anh ta cầm lấy chiếc nhẫn, đứng dưới ngọn đèn mà thả cho nó rơi. Tất nhiên là tìm lại được ngay (ở đó sáng lắm). Cứ như thế mười lần, hai mươi lần, một trăm lần, ... anh ta phát hiện ra quy luật: Khi rơi thì cái nhẫn nói chung chạy theo hướng nào. Bởi thế lúc vào góc nhà kho tối om, anh ta tìm ra ngay chiếc nhẫn. Mà không chỉ chiếc nhẫn ấy, nhà kho ấy, mà dù chiếc nhẫn khác, rơi ở nhà kho khác cũng tối om như vậy, thì đối với anh làm Toán, tìm nó cũng chẳng khó khăn gì! 10 Tạp chí Epsilon, Số 09, 06/2016 Các phương trình, các lý thuyết Toán học cũng như ngọn đèn của bác Tôm vậy. Có nó, người ta mới “làm Toán” được, tức là mới tìm ra quy luật của sự vật. Muốn trở về được với thực tiễn thì trước tiên phải biết rời xa thực tiễn, để không còn bị che lấp bởi cái rườm rà, không bản chất của đời thường. Ba trăm năm trước bác Tôm, Newton đã từng nói: “Không có gì gần với thực tiễn hơn là một lí thuyết đẹp!” 4. Ứng dụng Toán học Nhưng cái câu hỏi “Ích gì, Toán học?” vẫn cứ lởn vởn đâu đây, nhất là khi nhìn những nhà toán học hàng đầu nghiên cứu những thứ hoàn toàn “xa rời thực tế” , mà ngay cả bản thân họ cũng chưa biết mình sẽ đi đến đâu. Người ta thường hỏi nhà Toán học: Lí thuyết của anh ứng dụng vào đâu? Không phải lúc nào cũng có câu trả lời. Vào thế kỉ thứ III trước Công nguyên, nếu ai đó hỏi Apolonius rằng nghiên cứu các đường cônic (nhận được bằng cách cắt mặt nón bởi mặt phẳng) để làm gì, thì chắc Apolonius không trả lời được. Ông ta chỉ nghiên cứu các đường cônic vì thấy là chúng “đẹp”. Không chỉ Apolonius không thể trả lời, mà hơn chục thế kỉ sau cũng không ai trả lời được. Phải chờ đến Kepler và Newton, tức là 20 thế kỉ sau, người ta mới biết ông già Apolonius đã từng làm trò chơi với các quỹ đạo chuyển động của các hành tinh! Chính vì bị ám ảnh bởi các đường cônic ngay từ thuở ấu thơ mà Kepler đã nghi ngờ kết luận của những người đi trước về quỹ đạo tròn, và đưa ra giả thuyết quỹ đạo đó là đường ellip, với hai tiêu cự rất gần nhau. Giả thuyết đã được Newton chứng minh, với định luật vạn vật hấp dẫn. “Cái đẹp”, từ chỗ không biết để làm gì, đã tìm thấy một ứng dụng vào loại vĩ đại nhất trong lịch sử, sau hơn hai ngàn năm. Bác Tôm có lần nói: Đối với những người mở đường, đừng hỏi họ đi đâu, khi người ta biết mình đi đâu, người ta không đi được xa “quand on sait òu va, on va pas loin”. Thật thế, nếu anh định đi đến Thành phố Hồ Chí Minh thì chắc là anh cũng chỉ đi đến Cà Mau là cùng. Ngay như cái anh Armstrong, biết mình đi đến Mặt trăng thì cũng chỉ đến đó thôi, rồi về. Còn bác Tôm chẳng biết mình đi đâu, nên bác có thể đi xa hơn, đến tận sao Hỏa, hay những miền đất mới của khoa học. Và chúng ta, dù không đi xa được như bác Tôm, nhưng muốn ngày mai có bát cơm ngon, thì đừng quá sốt ruột nếu hôm nay chưa “ra ngô, ra khoai” gì! Còn nếu muốn “ra ngô, ra khoai” ngay thì có khi cả đời chỉ biết ăn ngô, ăn khoai! Vậy nhưng, nếu các lý thuyết Toán học đều phải cần đến 2000 năm sau mới có ứng dụng, thì câu hỏi “Ích gì, Toán học?” sẽ dễ nhận được câu trả lời là “Vô ích”! Dù “nhìn xa” đến mấy, người ta cũng khó nhìn đến tận ... 2000 năm sau! Nhưng Toán học đi vào thực tiễn với những con đường khác nhau. Có khi 2000 năm, có khi chỉ hai năm, thậm chí chỉ cần hai tháng! Ví dụ nổi tiếng là những hệ mật mã khoá công khai, như hệ mã RSA hay hệ mã dùng đường cong elliptic. Từ trang giấy của nhà nghiên cứu toán học đến ứng dụng vào cái điện thoại thông minh hay cái thẻ tín dụng gần như là tức thời. Không chỉ là những ứng dụng dễ nhìn thấy, Toán học giúp cho con người luôn hướng đến sự đơn giản trong tư duy. Tư duy Toán học chính là lối tư duy loại bỏ tất cả những gì không cần thiết, những gì rườm rà. Sự tối giản chính là một tiêu chuẩn của sự tối ưu, và nhiều khi, còn là tiêu chuẩn của cái đẹp. Một lần nữa, Toán học lại rất gần với Nghệ thuật. Nhà điêu khắc vĩ đại người Pháp Auguste Rodin (1840 − 1917) đã sáng tạo nên pho tượng bất hủ Le Penseur (Người suy tư), khắc họa hình ảnh một con người mà sự suy nghĩ căng thẳng hiện ra trên từng thớ thịt. Có 11 Tạp chí Epsilon, Số 09, 06/2016 người hỏi Rodin: “Làm thế nào mà ông có thể tạc nên pho tượng tuyệt vời đến vậy?”. Rodin trả lời: “Đơn giản thôi, tôi lấy một khối đá, và thấy cái gì thừa thì đẽo nó đi!”. Nhưng, tại sao sau tất cả những điều đã nói, vẫn tồn tại dai dẳng câu hỏi: ”Ích gì, Toán học”? Người ta hàng ngày dùng điện thoại di động để nói đủ thứ chuyện, đôi khi là để nói về cái sự vô ích của Toán học. Người ta hàng ngày dùng thẻ tín dụng để chuyển tiền, rút tiền. Nhưng sẽ không có điện thoại thông minh, không có thẻ tín dụng nếu không có mật mã khoá công khai, không có Toán học. Vậy nhưng người ta có thể vẫn rất ngại dùng tiền đó đầu tư cho Toán học, vì “Ích gì, Toán học?” Khi dùng điện thoại, khi rút tiền, không ai thấy “tích phân, vi phân, tổ hợp hay số học” trong đó. Nói khác đi, Toán học đã đến mức “trong suốt” đối với người sử dụng nó (tất nhiên chỉ khi đó nó mới được dùng cho tất cả mọi người). Xem ra, sự “trong suốt” đôi khi lại che khuất tầm nhìn hơn ngọn núi! Năm 1674 Mayow tìm thấy trong khí quyển một chất giúp cho sự sống. Năm 1773 Karl Scheele lần đầu tiên cô lập được chất khí đó. Antoine Lavoisier lặp lại thí nghiệm đó của Scheele và gọi đó là “oxygen”. Như vậy, oxy được “tìm ra” khá muộn. Tại sao? Vì nó trong suốt. Người ta hầu như không nhận thấy mình đang cần đến oxy. Và không chịu bỏ tiền ra “cho nó”. Phải đến khi con người nhìn thấy những hình ảnh sau đây: Đó là không khí ở Bắc Kinh. Nó không còn trong suốt nữa. Và người ta buộc phải nhìn thấy nó, buộc phải họp nhau ở Rio de Janeiro, ở Kyoto, ở Paris để bàn nhau tìm cách bỏ tiền ra làm cho nó trong suốt trở lại. Giá như người ta nhìn thấy sự cần thiết từ khi nó còn trong suốt! 5. “Tính” và “Toán” Một người bạn của bác Tôm, ông F.Hirzebruch, khi trả lời phỏng vấn của các nhà báo, trên cương vị là Chủ tịch đầu tiên của Hội Toán học Châu Âu, đã nói: “Người ta thường hay nhấn mạnh vai 12 Tạp chí Epsilon, Số 09, 06/2016 trò của Toán học trong phát triển công nghệ, nhưng tôi nghĩ rằng, sẽ đến lúc công nghệ phát triển để giải phóng con người, cho họ thời gian quay về với thơ ca, âm nhạc và Toán học”. Phải chăng, Hirzebruch muốn ám chỉ rằng, trong Toán học có hai phần: “Tính” và “toán”. Nếu như tính rất cần thiết cho công nghệ, thì Toán, ngoài chức năng phát triển phần tính ra, còn góp phần làm nên Con Người, cũng giống như âm nhạc, nghệ thuật và thơ ca. Nhưng có thể 5 năm nữa, trong dịp kỷ niệm 10 năm của VIASM, chúng ta lại sẽ phải bàn về câu hỏi: “Ích gì, Toán học?” Phải chăng đó là câu hỏi vĩnh cửu, song hành với Toán học từ khi nó ra đời? Cũng như câu hỏi tương tự cũng song hành cùng Nghệ thuật và Thơ ca. 13 Tạp chí Epsilon, Số 09, 06/2016 14 HÀM MOEBIUS VÀ ĐỊNH LÝ PHẦN DƯ TRUNG HOA Phùng Hồ Hải (Viện Toán học Việt Nam) Câu lạc bộ (CLB) Toán học Viện toán học được thành lập cách đây 5 năm. Đầu tiên CLB sinh hoạt sáng chủ nhật mỗi tuần với các bài giảng toán của các GS, TS, các thầy giáo chuyên toán dành cho học sinh THPT. Sau đó để tạo điều kiện cho các học sinh tỉnh xa, CLB đã tổ chức những đợt giảng dài ngày dưới hình thức trường đông, trường hè, trường xuân. Gần đây CLB toán học Viện toán học đã tổ chức thêm các lớp học dành cho học sinh THCS với mục tiêu định hướng và tạo nguồn. Epsilon xin giới thiệu với bạn đọc bài viết của GS Phùng Hồ Hải kể lại buổi dạy ngẫu hứng của mình vào chiều thứ ba 7/6/2016 vừa qua. Bài viết được lấy từ trang facebook cá nhân của giáo sư Phùng Hồ Hải. Để hiểu nội dung bài viết, ta cần đến định nghĩa của hàm Moebius. Hàm Moebius µ(n) được xác định trên tập hợp các số nguyên dương và nhận giá trị trong tập hợp {−1, 0, 1}. Ta có µ(1) = 1. Khi n > 1, µ(n) sẽ bằng 0 nếu n có ít nhất một ước chính phương lớn hơn 1. Trong trường hợp n không có ước chính phương (lớn hơn 1) thì với n = p1p2 · · · pr là tích của các số nguyên tố phân biệt ta sẽ có µ(n) = (−1)r. Ví dụ ta có bảng giá trị của hàm Moebius với n 6 15. n 1 2 3 4 5 6 7 8 9 10 11 12 13 14 15 µ(n) 1 −1 −1 0 −1 1 −1 0 0 1 −1 0 −1 1 1 Lâu rồi không report về Math Circle. Hôm nay tôi đã có một buổi dạy khá ngẫu hứng. Hàm Moebius đã được nhắc tới vài lần. Hôm nay tôi muốn thuyết phục các bạn trong lớp rằng hàm này bằng 0 tại “phần lớn” các số tự nhiên. Nói phải có sách, mách phải có chứng. Ta sẽ thử liệt kê xem hàm sẽ triệt tiêu tại những giá trị nào. Ta biết rằng hàm sẽ triệt tiêu tại n nếu n chia hết cho bình phương một số nguyên tố. Như vậy trong 10 số tự nhiên đầu tiên, hàm sẽ triệt tiêu tại 4, 8, 9 - ba số. Trong 10 số tự nhiên tiếp theo hàm sẽ triệt tiêu tại 12, 16, 18, 20 - bốn số. Tình hình sau đó kém đi một chút: 32, 36, 40. Rồi lại khá lến: 44, 45, 48, 49, 50. Dù sao thì các bạn trong lớp cũng thấy rằng tình hình có chiều hướng tốt lên, nghĩa là càng ngày thì tỷ lệ các số mà tại đó hàm triệt tiêu càng tăng. Đặc biệt ta thấy xuất hiện ba số liên tiếp mà tại đó hàm triệt tiêu: 48, 49, 50. - Nếu chúng ta đi tiếp thì chúng ta có thể tìm thấy 10 số tự nhiên liên tiếp mà tại đó hàm Moebius triệt tiêu. Thầy tuyên bố. Khẳng định có vẻ làm cho các bạn học sinh ngạc nhiên. - Chúng ta đã thấy ở trên có ba số liên tiếp. Bạn nào có thể chứng minh rằng tồn tại bốn số liên tiếp mà tại đó hàm triệt tiêu? Câu hỏi có vẻ hơi khó. - Hay là thế này: Chứng minh rằng tồn tại vô hạn cặp hai số tự nhiên liên tiếp mà tại đó hàm Moebius triệt tiêu. 15 Tạp chí Epsilon, Số 09, 06/2016 Ngay lập tức có lời giải: xét hai số a2 − 1 và a2, hàm hiển nhiên triệt tiêu tại a2, chỉ cần chọn a lẻ thì a2 − 1 chia hết cho 4. - Thế ba số thì sao, liệu có tồn tại vô hạn bộ ba số tự nhiên liên tiếp mà hàm Moebius triệt tiêu tại đó? Sau một lúc trao đổi thì các bạn đề xuất lời giải thế này: xét số a3 − 1, a3, a3 + 1. Hàm Moebius tất nhiên triệt tiêu tại a3. Ta sẽ tìm a sao cho a3 − 1 chia hết cho 4 còn a3 + 1 chia hết cho 9. Tuy nhiên lời giải chưa chặt chẽ. Thầy giải thích cho các bạn rằng a3 − 1 chia hết cho 4 khi và chỉ khi a − 1 chia hết cho 4 còn a3 + 1 chia hết cho 9 khi và chỉ khi a + 1 chia hết cho 3. Vậy bài toán là những số a nào thỏa mãn các điều kiện này? Đây là một bài toán dễ đối với các bạn, câu trả lời có ngay: a chia 12 dư 5, hay a = 12k + 5. - Bài toán tìm a để a đồng dư với 3 mod 4 và đồng dư với 2 mod 3 chính là bài toán “giải hệ phương trình đồng dư bậc nhất”, thầy giải thích cho các bạn. Mọi người rất phấn khích. - Bây giờ ta hãy xem nguyên tắc giải hệ như thế nào: Trước hết ta tìm một “nghiệm riêng”, chính là số 5, từ đó ta chỉ cần cộng thêm “nghiệm tổng quát”, là số 12 - bội chung nhỏ nhất của 3 và 4. Quá dễ hiểu đối với các bạn học sinh. - Bây giờ ta hãy xét một hệ 3 phương trình xem sao. Chẳng hạn: a đồng dư với 1 mod 3, đồng dư với 2 mod 4 và đồng dư với 4 mod 5. a = 34 + 60k, chỉ trong vài phút các bạn đã có câu trả lời. - Chúng ta có thể giải hệ trên “dần dần”, xét hai phương trình đầu tiên ta sẽ có nghiệm: a đồng dư với 10 mod 12, sau đó ta lại xét phương trình này cùng với phương trình thứ ba, a đồng dư với 4 mod 5, để suy ra nghiệm bằng phương pháp tương tự. - Bằng cách này ta chứng minh được Định lý phần dư Trung Hoa: Mọi hệ phương trình đồng dư bậc nhất với các modun đôi một nguyên tố cùng nhau đều có nghiệm. Chỉ cần xét dần dần, mỗi lần hai phương trình. - Bạn nào biết tại sao hệ hai phương trình đồng dư bậc nhất với modun nguyên tố cùng nhau luôn có nghiệm? - Sử dụng hệ thức Bezout, một bạn trả lời ngay lập tức. Thế là định lý phần dư Trung Hoa được chứng minh, ít nhất là “về nguyên tắc”. Trở lại bài toán ban đầu, phương pháp sử dụng a3rồi thêm bớt 1 không thể cho ta lời giải cho câu hỏi: Tìm 10 số tự nhiên liên tiếp mà tại đó hàm Moebius triệt tiêu. Đó là vì phương pháp xây dựng của ta không phù hợp. Chúng ta có thể sử dụng Định lý phần dư Trung Hoa để có một cách xây dựng đơn giản hơn nhiều. Chỉ cần có 5 phút để một bạn có lời giải hoàn chỉnh cho bài toán này. 16 DẪN NHẬP VỀ HÀM ZETA RIEMANN VÀ PHÉP BIẾN ĐỔI MELLIN Ngô Bảo Châu (Đại học Chicago) Đọc lịch sử toán học ta thấy rằng trước đây thư tín là một phương tiện trao đổi thông tin toán học rất quan trọng. Rất nhiều những ý tưởng, giả thuyết, dự đoán và cả những chứng minh hay phản ví dụ đã được trình bày lần đầu tiên ở trong những bức thư chứ không phải là ở trong các tạp chí và cuốn sách. Newton, Leibniz, Fermat, Euler, Gauss, Jacobi, Bernoulli, Mersenne, ... đều có những “công bố” ở dạng thư tín. Ngày nay, với những phương tiện giao tiếp hiện đại như blog, trang wordpress, archiv . . . ta có nhiều cách để “công bố” các ý tưởng hay tiền ấn phẩm của mình. Tuy nhiên, trao đổi học thuật bằng thư tín, email vẫn là một hình thức thông dụng và hiệu quả. Thông tin qua email có thể sẽ không hình thức và chặt chẽ như những bài báo, những cuốn sách, có nhiều ý tưởng còn thô ráp, nôm na, nhưng chính điều đó làm độc giả dễ đọc hơn, gần gũi hơn, dễ hiểu hơn. Chúng tôi xin giới thiệu với bạn đọc lá thư của GS Ngô Bảo Châu gửi cho GS Phùng Hồ Hải nhân việc GS Hải muốn tổ chức một khóa học về hàm zeta và phép biến đổi Mellin để chuẩn bị cho chuỗi bài giảng về lý thuyết biểu diễn của GS Ngô Bảo Châu và GS Phạm Hữu Tiệp vào tháng 8/2016. Dịch bởi Nguyễn Vũ Duy Linh và Trần Nam Dũng hiệu đính. From: Đại Học CHICAGO Khoa Toán 5734 University Avenue, Chicago IL 60637 Ngô Bảo Châu To: GS. Phùng Hồ Hải Viện Toán học Việt Nam 18 Hoàng Quốc Việt, Hà Nội Ngày 5 tháng 5 năm 2016. Hải thân mến, Tổ chức một seminar học tập về hàm zeta có vẻ là một ý tưởng tốt. Tôi thiết nghĩ nên mở đầu bằng một vài vấn đề rất căn bản của lý thuyết số giải tích. Chúng ta hãy xem xét một hàm số học tức là đơn thuần một dãy số a1, a2, . . . được định nghĩa theo kiểu số học nào đó. Đây là một khái niệm trừu tượng và người ta không thể làm được gì ở mức độ trừu tượng như vậy. Trên một phương diện khác, ta có rất nhiều ví dụ cụ thể: 1. an = 1, 17 Tạp chí Epsilon, Số 09, 06/2016 2. an = µ(n) là hàm Moebius, 3. an = d(n) là số những ước số của n, 4. an = r(n) là số cách biểu diễn n như là tổng của hai bình phương, 5. an = 1 nếu n nguyên tố và bằng 0 nếu ngược lại, 6. an = log(p) nếu n là lũy thừa của p và 0 nếu ngược lại, ... Mặc dầu những dãy số này rất bất thường, ngoại trừ dãy đầu tiên, tổng Cesaro a1 + a2 + · · · + an có tính tiệm cận nào đó. Chẳng hạn chúng ta xác định tính tiệm cận của dãy thứ 5, chúng ta thử tính số lượng số nguyên tố nhỏ hơn một số nào đó. Ước lượng tổng Cesaro của d(n) và r(n) là những vấn đề sơ cấp cổ điển trong lý thuyết số. Đối với r(n), đó là những bài toán về hình tròn Gauss: Có bao nhiêu điểm nguyên nằm trong hình tròn khi bán kính tiến về ∞. Bằng những lý luận sơ cấp như Gauss đã làm, chúng ta có X n α nào đó. Chẳng hạn chuỗi Dirichlet tương ứng với dãy an = 1 chính là Riemann ς-function ς(s, a) = Pn n−s hội tụ trên miền <(s) > 1. Có một vài thao tác mà chúng ta nên làm quen khi làm việc với chuỗi Dirichlet. Chẳng hạn, trong bốn ví dụ đầu tiên, an có tính chất nhân tính: amn = aman với (m; n) = 1. Chuỗi Dirichlet tương ứng có thể phân tích thành nhân tử thành tích Euler tức là tích của các thừa số đánh số bằng số nguyên tố. Chẳng hạn như: ς(s) = Y p (1 − p−s)−1. (0.2) Một nhận xét khác nhưng theo lối tương tự là chuỗi Dirichlet tương ứng với tích chập của các hàm số học. Chẳng hạn nhưX n d(n)ann−s = ς(s)2, (0.3) vàX n µ(n)ann−s = ς(s)−1. (0.4) 18 Tạp chí Epsilon, Số 09, 06/2016 Những hằng đẳng thức này là hiển nhiên khi xem xét như là chuỗi Dirichlet hình thức. Như là các hàm phức, chúng có thể được chứng minh dễ dàng trên miền mà chuỗi Dirichlet hội tụ tuyệt đối. Tất cả những điều này được giải thích hoàn hảo trong chương 2 của quyển “Nhập môn lý thuyết số giải tích” của Apostol. Một trong những hướng chính của lý thuyết số giải tích cổ điển là dẫn ra một ước lượng tiệm cận của tổng Cesaro từ những cực của chuỗi Dirichlet. Đó chính là nội dung của định lý Tauberian. Một vấn đề khó chịu là có nhiều phiên bản của định lý Tauberian. Phiên bản đơn giản nhất có thể là định lý Wiener-Ikehara’s được giải thích trong chương 11 của Chandrasekharan chẳng hạn. Trong bất cứ trường hợp nào, định lý Tauberian cho ta biết một điều đại loại như sau: Giả sử rằng D(s, a) = Pnann−scó thể thác triển phân hình vượt qua đường thẳng <(s) = 1 − ε. Giả sử rằng nó chỉ có một cực cấp 2 tại 2 và một cực cấp 1 tại 1. Khi đó biểu thức tiệm cận sẽ có dạng: X n 1, điều này được chứng minh bởi Hadamard cả trăm năm về trước. Nếu anh có đủ thời gian và can đảm, anh hãy nghiên cứu lại tất cả những thứ đó cũng như định lý Dirichlet về số nguyên tố trong cấp số cộng. Đó là chỗ mà chúng ta cần một luật thuận nghịch nào đó. Nhưng anh không cần bắt đầu với luật thuận nghịch vì thật sự ra nó không phải là hướng chính trong sự phát triển ban đầu của hàm zeta, ít ra theo ý kiến của tôi. Tất nhiên về sau luật thuận nghịch sẽ trộn lẫn rất nhiều vào lý thuyết của hàm zeta cũng như trong chương trình của Langlands, nhưng mỗi lúc học một thứ thì vẫn dễ dàng hơn. Chúc anh mọi điều tốt lành Châu 21 Tạp chí Epsilon, Số 09, 06/2016 22 GIẢI NOBEL CỦA EINSTEIN HAY LÀ SÓNG ĐIỆN THOẠI CÓ GÂY UNG THƯ HAY KHÔNG (Đàm Thanh Sơn - ĐH Chicago) Bài viết được tham khảo từ blog của giáo sư Đàm Thanh Sơn. Albert Einstein có lẽ là nhà vật lý nổi tiếng nhất từ trước đến nay. Công chúng thường biết đến ông ta như người khám phá ra thuyết tương đối, làm thay đổi quan niệm của chúng ta về không gian và thời gian. Thuyết tương đối bao gồm thuyết tương đối hẹp, được Einstein tìm ra năm 1905 và thuyết tương đối rộng được ông ta tìm ra 10 năm sau. Tuy nhiên có thể không phải ai cũng biết là giải thưởng Nobel về vật lý năm 1921 được trao cho Einstein vì một khám phá khác của ông: Hiệu ứng quang điện. Đây là công trình Einstein viết cũng vào năm 1905, cùng năm với công trình về thuyết tương đối hẹp và một công trình nữa về chuyển động Brown. Hiệu ứng quang điện là đóng góp lớn nhất của Einstein vào thuyết lượng tử, lý thuyết mà sau này được Bohr, Heisenberg, Schrodinger và nhiều người khác phát triển lên nhưng lại bị Einstein nghi ngờ ¨ đến cuối đời. Hiệu ứng quang điện là hiện tượng khi ta chiếu ánh sáng vào một tấm kim loại thì thỉnh thoảng điện tử bị bứt ra khỏi kim loại. Ta có thể đoán là ánh sáng càng mạnh thì càng nhiều điện tử bị bứt ra. Phán đoán này hoá ra là không hoàn toàn đúng: Có những nguồn ánh sáng rất mạnh không gây ra hiệu ứng quang điện, nhưng có những nguồn yếu hơn lại gây ra hiệu ứng này. Thực nghiệm cho thấy rằng hiệu ứng quang điện phụ thuộc vào tần số của ánh sáng. Ví dụ, với cùng một mẫu kim loại, ánh sáng đỏ hoặc tia hồng ngoại không gây ra hiệu ứng nhưng ánh sáng tím hoặc cực tím lại có tác dụng. 23 Tạp chí Epsilon, Số 09, 06/2016 Einstein giải thích điều này bằng cách áp dụng và mở rộng giả thuyết lượng tử của Planck. Einstein giả thuyết rằng ánh sáng bao gồm các hạt photon, mỗi hạt mang một năng lượng tỉ lệ thuận với tần số của ánh sáng. E = hv. Ở đây E là năng lượng của hạt photon, v là tần số của ánh sáng, và h là hằng số Planck. Công thức trên có tên là công thức Planck, công thức mà theo tôi đáng lẽ ra phải nổi tiếng hơn công thức E = mc2. Hiệu ứng quang điện là quá trình một hạt photon truyền năng lượng cho một hạt điện tử. Để bứt một điện tử ra khỏi mảnh kim loại ta cần một năng lượng tối thiểu nhất định, ta gọi là ∆. Như vậy chỉ khi v > ∆hánh sáng mới có thể bứt được điện tử ra khỏi khối kim loại. Nếu v < ∆hthì nguồn sáng có mạnh thế nào cũng không có photon đủ năng lượng để gây ra hiệu ứng quang điện. Bạn có thể hỏi liệu có khi nào hai hạt photon, hoặc nhiều hơn, cùng hợp sức để bứt ra một điện tử hay không. Điều này về nguyên tắc có thể xảy ra, nhưng xác suất rất thấp, có thể bỏ qua. Hiệu ứng quang điện có liên quan trực tiếp đến một câu hỏi hay được đặt ra hiện nay: Điện thoại di động có gây tác hại cho sức khoẻ hay không? Một trong những điều làm nhiều người lo lắng là khả năng gây ung thư của sóng điện thoại (ví dụ xem bài này). Nhiều người còn nói là sóng điện từ trong lò vi sóng cũng có thể gây ra ung thư. Nếu ta nhớ lại công thức E = hv của Einstein thì ta sẽ thấy những lo lắng này không có cơ sở. Đó là do tần số sóng của các thiết bị điện tử quá thấp để có thể gây ra những biến đổi của phân tử ADN. Tần số sóng trong lò vi sóng là 2500 MHz, tần số của điện thoại di động là 800 MHz hay 1900 MHz. Hằng số Planck là 4 × 10−9eV/MHz, như vậy 2500 MHz tương đương với năng lượng 10 phần triệu eV, trong khi các quá trình hoá học hay sinh hoá cần năng lượng cỡ ít nhất 0.1 eV, nếu không phải là 1 eV. Sự chênh lệch đến 10 − 100 nghìn lần giữa hai cỡ năng lượng làm cho lò vi sóng hay điện thoại di động không thể làm biến đổi gien của miếng thịt để trong lò hay cơ thể chúng ta. (Tia cực tím thì lại khác, vì tần số của tia cực tím cao hơn tần số của điện thoại di động đến cả triệu lần, nên nó có đủ năng lượng để gây tác hại cho tế bào). Tất nhiên là điện thoại di động hay các thiết bị điện tử có thể có những tác hại khác, ví dụ cho tâm lý hay là giấc ngủ của người dùng, nhưng chúng ở ngoài khuôn khổ của bài viết này. 24 HỆ MẬT Mà KHÓA CÔNG KHAI DỰA TRÊN ĐƯỜNG CONG ELLIPTIC - TỔNG QUAN VỀ HỆ MẬT Mà KHÓA CÔNG KHAI Đặng Minh Tuấn (Vietkey) Trong số này, Epsilon trân trọng gửi đến độc giả phần đầu tiên của chuyên đề về hệ mật mã khóa công khai dựa trên đường cong Elliptic của tác giả Đặng Minh Tuấn. Phần sau của chuyên đề sẽ được chuyển tải ở tạp chí số 10. Sau đây là toàn văn của lời mở đầu và chương 1 của loạt chuyên đề. LỜI MỞ ĐẦU Tháng 3 năm 2016, Bộ Ngoại Giao Hoa Kỳ, đứng đầu là bộ trưởng John Kerry, đã dẫn một đoàn đại biểu tới các nước ASEAN trong đó có Việt Nam để thảo luận về phát triển Fintech và đặc biệt là về công nghệ Blockchain1.Tháng 9 năm 2015, Ủy ban giao dịch hàng hoá tương lai Mỹ công bố, Bitcoin đã chính thức được đưa vào danh sách hàng hóa được phép giao dịch tại Mỹ2.Công nghệ Blockchain và Bitcoin là công nghệ tiền số ra đời năm 2009 và ngày càng có nhiều quốc gia và các tổ chức, doanh nghiệp cho phép lưu hành và thanh toán bằng loại tiền số này trong không gian mạng Internet toàn cầu. Tháng 4-2016, giá trị thương mại của Bitcoin đã lên đến 6.5 tỷ USD 3. Nền tảng cơ sở của Bitcoin chính là lý thuyết về mật mã mà cụ thể ở đây là hàm băm và lý thuyết về chữ ký số dựa trên Hệ mật đường cong Elliptic (ECC). Bên cạnh việc sử dụng trong tiền số Bitcoin4, ECC còn được ứng dụng rất nhiều trong thực tiễn ngành Công nghệ thông tin [1]. Các trang Web bảo mật https (http-secure) thường được dùng trong thanh toán điện tử hay ứng dụng riêng tư như gmail đều sử dụng các giao thức TLS (Transport Layer Security) mà trước đó là 1B. Cohen. (April 12, 2016) U.S. State Department Recommends Development of Blockchain and Dis tributed Ledgers to International Partners. [Online]. Available: http://www.nasdaq.com/article/us-state-department recommendsdevelopment-of-blockchain-and-distributed-ledgers-to-international-partnerscm605334. 2(21/09/2015) Bitcoin chính thức được Mỹ công nhận là hàng hoá. [Online]. Available: http://vnreview.vn/tin tuc-kinh-doanh/-/view-content/content/1654484/bitcoin-chinh-thuc-duoc-My-cong-nhan-la-hang-hoa. 3(15/4/2016) https://markets.blockchain.info/ 4Địa chỉ ví Bitcoin được tính dựa trên khóa công khai của ECC với hàm băm bảo mật có độ dài 256-bit SHA256 và thuật toán Base58Encode dùng để chuyển số thành dạng 56 ký tự, RIPEMD (RACE Integrity Primitives Evaluation Message Digest) là một họ hàm băm bảo mật khác: Version = 1(byte) KeyHash = Version + RIPEMD(SHA256(PublicKeyEC)) CheckSum = SHA256(SHA256(KeyHash)) BitcoinAddress = Base58Encode(KeyHash + CheckSum) 25 Tạp chí Epsilon, Số 09, 06/2016 SSL (Secure Socket Layer). Trong các giao thức này ECC được sử dụng để trao đổi khóa phiên. Các giao dịch remote access được sử dụng rất nhiều trong thế giới Unix, Linux là SSH (Secure SHell) cũng sử dụng ECC để trao đổi khóa. Ưu điểm của hệ mật sử dụng đường cong Elliptic (ECC) là có độ dài khóa nhỏ (160 bit tương đương với khóa độ dài 1024 Bit trong hệ mật RSA), do sử dụng độ dài khóa nhỏ nên tài nguyên phục vụ cho ECC thường nhỏ hơn rất nhiều, bên cạnh đó hiệu năng tính toán cũng được nâng cao rõ rệt. Hiện nay ECC đang là xu thế để thay thế RSA. Cơ sở toán học của hệ mật ECC là nhóm giao hoán Abel các điểm nằm trên đường cong Elliptic. Ngoài việc đường cong Elliptic là cơ sở cho hệ mật ECC, hệ mật ID-Based, đường cong Elliptic (EC) còn là công cụ hữu hiệu để phân tích số nguyên ra thừa cố nguyên tố [2, 3, 4], hoặc dùng để kiểm tra tính nguyên tố của số nguyên [3]. EC cũng là cơ sở để chứng minh định lý Fermat nổi tiếng đã tồn tại nhiều trăm năm qua. Đường cong Elliptic là một trường hợp đặc biệt của phương trình Diophant. Lý thuyết về đường cong Elliptic (EC) rất phong phú và đồ sộ. Trong [5] tác giả Serge Lang đã phát biểu về phương diện học thuật: “Có thể viết vô tận về đường cong Elliptic”. Các lý thuyết và khái niệm liên quan tới EC có thể liệt kê một số như dưới đây: • Lý thuyết nhóm, vành, trường trong đại số trừu tượng [3, 6]; • Đa tạp Affine, đa tạp Jacobian và đa tạp xạ ảnh trong hình học đại số [7, 8]; • Điểm Torsion, Divisor, cặp song tuyến tính Weil, Tate-Lichtenbaum [3]; • Lý thuyết trường Galois, tự đồng cấu-ánh xạ Frobenius [3, 9]; • Lý thuyết Baker-Feldman, Baker-Tijdeman và lý thuyết Kummer [5]; • Số p-adic, Isogenies, hàm Sigma và hàm Zeta [5, 7, 10, 3, 4]; • Nhóm đối đồng điều, đối đồng điều Galois và đối đồng điều phi giao hoán (Topo đại số) [8, 4]; • Nhóm Mordell–Weil, Selmer và nhóm Shafarevich–Tate [8, 11]; • Phương pháp hình học và Tựa tuyến tính (Quasilinear) [12]. Với ý nghĩa to lớn cả về thực tiễn và học thuật, EC là nền tảng toán học quan trọng trong đại số hiện đại cũng như lý thuyết mật mã hiện đại. EC cũng là nền tảng quan trọng trong chính phủ điện tử và thương mại điện tử. Chính vì những điều này mà Chuyên đề “Hệ mật mã khóa công khai dựa trên đường cong Elliptic” được lựa chọn để trình bày báo cáo. Với khối lượng kiến thức và khái niệm đồ sộ như đã liệt kê ở trên việc nghiên cứu và đào sâu về đường cong Elliptic gặp không ít khó khăn cho những người làm Công nghệ thông tin (CNTT) mà toán học không phải là chuyên môn chính. Mục tiêu của chuyên đề này là tổng hợp những khái niệm và kiến thức cơ bản nhất của EC liên quan đến cơ sở toán học của Hệ mật dựa trên đường cong Elliptic. Đồng thời người viết cũng chứng minh lại một số định lý và bổ đề theo cách dễ hiểu hơn, tránh dùng đến các khái niệm quá phức tạp và xa lạ với chuyên ngành CNTT. Các phép toán của đường cong Elliptic được trình bày trong báo cáo phần lớn được cài đặt bằng phần cứng sử dụng công nghệ FPGA (Field-Programmable Gate Array) của Xilinx trong khuôn khổ Đề tài cấp Nhà nước KC01-18 (đã được nghiệm thu trong năm 2014) do người viết báo cáo làm chủ nghiệm đề tài, kết quả được công bố tại [13]. 26 Tạp chí Epsilon, Số 09, 06/2016 Phạm vi của chuyên đề cũng được giới hạn với những khái niệm và lý thuyết đủ cho các ứng dụng cơ bản của EC, các phát triển của EC thành hệ mật ID-Based, hoặc các ứng dụng về chữ ký số tập thể, chữ ký số nhóm, chữ ký ngưỡng, chứ ký ủy nhiệm, chứ ký số mù sẽ không được đề cập đến trong khuôn khổ của báo cáo này. Báo cáo chuyên đề được kết cấu thành 02 chương, chương 1 trình bày các khái niệm, định nghĩa cơ bản về đường cong Elliptic (Phương trình của EC, nhóm cộng Abel các điểm trên đường cong, chứng minh định lý về nhóm...). Chương 2 trình bày về Hệ mật dựa trên đường cong Elliptic và một số ứng dụng trong mã hóa, xác thực chữ ký số, trao đổi khóa dự trên bài toán khó Logarithm rời rạc. Ký hiệu N tập hợp số tự nhiên N∗tập hợp số tự nhiên khác 0 Z trường số nguyên Q trường số hữu tỉ char(K) đặc số của trường K ≡ dấu đồng dư ∞ dương vô cùng (tương đương với +∞) gcd ước số chung lớn nhất deg bậc của đa thức det định thức RSA Hệ mã công khai dựa trên bài toán phân tích ra thừa số nguyên tố do Rivest-Shamir-Adleman phát triển EC Đường cong Elliptic (Elliptic Curve) ECC Hệ mật dựa trên đường cong Elliptic (Elliptic Curve Cryptography) ECDLP Elliptic Curve Logarithm Problem ECDH Thuật toán Elliptic Curve Diffie–Hellman ECDSA The Elliptic Curve Digital Signature Algorithm ECIES The Elliptic Curve Integrated Encryption System ECMQV Elliptic Curve Menezes–Qu–Vanstone protocol 1. Tổng quan về đường cong Elliptic Năm 250 sau Công nguyên, Diophant khi giải bài toán tìm số tầng của tháp các quả cầu mà khi trải ra mặt đất có thể xếp thành một hình vuông đã dẫn đến giải phương trình (y là số quả cầu trên 1 cạnh hình vuông; x là số tầng của tháp): y2 = 12 + 22 + 32 + · · · + x2 =x(x + 1)(2x + 1) 6 Phương trình y2 = x(x + 1)(2x + 1)/6 là một dạng của đường cong Elliptic. Năm 1637, nhà toán học và vật lý học người Pháp Pierre de Fermat công bố định lý Fermat cuối cùng khi viết trên lề bản copy công trình của Diophant: Phương trình sau đây là vô nghiệm: xn + yn = zn, n > 2 27 Tạp chí Epsilon, Số 09, 06/2016 Hơn ba thế kỷ, đã có rất nhiều nhà toán học cố gắng chứng minh định lý này xong đều thất bại, mãi cho đến năm 1994, Andrew Wiles, giáo sư trường Princeton đã gây một tiếng vang lớn trong cộng đồng toán học thế giới vào thời điểm đó khi sử dụng đường cong Elliptic có dạng y2 = x(x − an)(x + bn) cùng với lý thuyết về Modul để chứng minh định lý Fermat cuối cùng. Năm 1987, Trong [14], Lenstra đề xuất thuật toán phân tích số nguyên ra thừa số nguyên tố sử dụng đường cong Elliptic, đó là thuật toán tương đối nhanh, chạy với thời gian dưới hàm mũ và là thuật toán nhanh thứ 3 trong việc phân tích ra thừa số nguyên tố, sau phương pháp sàng đa thức toàn phương và phương pháp sàng trường số tổng quát. Trong lĩnh vực mật mã, vào năm 1985, Victor S. Miller công bố bài báo đầu tiên về ứng dụng đường cong EC trong mật mã “Use of Elliptic Curves in Cryptography” [15] và sau đó là Neal Koblitz với “Elliptic curve cryptosystem” [16] vào năm 1987. Từ đó cho đến nay đã có rất nhiều công bố nghiên cứu về EC về lý thuyết và trong thực tiễn càng ngày ứng dụng ECC càng được sử dụng rộng rãi, và đã được đưa thành các tiêu chuẩn. Một số tiêu chuẩn liên quan đến đường cong Elliptic: IEEE 1363: Tiêu chuẩn này bao gồm gần như tất cả các thuật toán về các hệ khóa công khai trong đó có ECDH, ECDSA, ECMQV và ECIES. Trong phần phụ lục có cả các thuật toán cơ bản về lý thuyết số liên quan đến hệ mật khóa công khai. ANSI X9.62 và X9.63: Các chuẩn này tập trung vào đường cong Elliptic và cụ thể về ECDSA trong X9.62 và ECDH, ECMQV và ECIES trong X9.63. Các chuẩn này cũng xác định khuôn dạng các dữ liệu và danh mục các đường cong khuyến cáo sử dụng. FIPS 186.2: Tiêu chuẩn của NIST cho chữ ký số, mô tả chi tiết về thuật toán DSA algorithm. SECG: Là tiêu chuẩn được biên soạn bởi nhóm các doanh nghiệp dẫn dắt bởi công ty Certicom, gần như là ánh xạ của các chuẩn ANSI nhưng được tiếp cận trên môi trường Web từ Website http://www.secg.org/ ISO 15946-2: Tiêu chuẩn mô tả về ECDSA và ECIES (còn được gọi là ECIES-KEM). RFC 3278: “Use of Elliptic Curve Cryptography (ECC) Algo rithms in Cryptographic Message Syntax (CMS)” là khuyến nghị sử dụng thuật toán ECC trong mã hóa thông điệp văn bản. 2. Phương trình Weierstraß của đường cong Elliptic Trong tài liệu này, đa phần số các đường cong Elliptic sẽ được nghiên cứu dưới dạng sau: y2 = x3 + Ax + B, (2.1) Trong đó A và B là các hằng số. Các giá trị của x, y, A, B thường là các giá trị trên một trường nào đó, ví dụ như R (số thực), Q (số hữu tỷ), C (số phức), hoặc trường hữu hạn Fq, với q = pn trong đó p là số nguyên tố với n = 1. Nếu K là một trường có a, b ∈ K, khi đó ta nói đường cong Elliptic được định nghĩa trên trường K. Điểm (x, y) trên đường cong Elliptic với (x, y) ∈ K 28 Tạp chí Epsilon, Số 09, 06/2016 được gọi là điểm K–Hữu tỷ. Dạng tổng quát phương trình Weierstrass của đường cong Elliptic sẽ được biểu diễn dưới dạng: y2 + a1xy + a3y = x3 + a2x2 + a4x + a6, (2.2) Trong đó a1, · · · , a6 là các hằng số. Dạng (2.2) thường được sử dụng với các trường K có đặc số chap(K) bằng 2 hoặc 3. Khi K có chap(K) khác 2 có thể biến đổi (2.2) thành dạng sau: y +a1x2+a32 2= x3 + a2 +a214 x2 + a4 +a1a3 2 x + a23 4+ a6 , Có thể viết lại như sau: y21 = x3 + a02x2 + a04x + a06, Với y1 = y + a1x/2 + a3/2 và với các hằng số a02, a04, a06. Khi K có chap(K) khác 3 có thể dùng phép thế x1 = x + a02/3 và ta có: y21 = x31 + Ax + B, Trong đó A, B là các hằng số nào đó. Đường cong (2.1) có định thức ∆ = −16(4A3 + 27B). Đường cong này sẽ suy biến và không có đủ 3 nghiệm phân biệt khi ∆ = 0, trong tài liệu này chúng ta chỉ xét các đường cong có ∆ 6= 0. 3. Cộng các điểm trên đường cong Elliptic Xét hai điểm P1 = (x1, y1) và P2 = (x2, y2) trên đường cong Elliptic E : y2 + a1xy + a3y = x3 + a2x2 + a4x + a6. Phép cộng giữa hai điểm trên đường cong E được định nghĩa như sau: P3(x3, y3) = P1(x1, y1) + P2(x2, y2) (3.1) Trong đó P3(x3, y3) = −P03(x3, y03), điểm P03(x3, y03) là giao điểm của đường cong E và đường thẳng đi qua P1 và P2. Vì 2 điểm P3(x3, y3) và −P03(x3, y03) đều nằm trên đường cong E nên (x3, y3) và (x3, y03) phải thỏa mãn phương trình (2.2). Công thức để tính các giá trị (x3, y3) sẽ được chứng minh ở dưới đây. Trong các các tài liệu cơ bản và nâng cao được tham chiếu nhiều về đường cong Elliptic như [3, 7, 8] người viết vẫn chưa thỏa mãn với các dẫn dắt và chứng minh công thức tổng quát cho các giá trị (x3, y3), do đó các công thức này sẽ được chứng minh chi tiết trong tài liệu này. Đường thẳng đi qua 2 điểm P1 và P2 có phương trình là: y = λx + µ (3.2) Trong đó λ là hệ số góc của đường thẳng đi qua P1, P2. Ta có: y1 = λx1 + µ (3.3) y2 = λx2 + µ (3.4) y03 = λx3 + µ (3.5) 29 Tạp chí Epsilon, Số 09, 06/2016 Hình 6.1: Phép cộng trên đường cong Elliptic 3.1. Trường hợp 2 điểm không trùng nhau P1 6= P2 Từ (3.3) và (3.4) suy ra: y1 − y2 = λ(x1 − x2), khi P1 6= P2, nghĩa là x1 6= x2 ta có công thức: λ =y1 − y2 x1 − x2(3.6) µ = y1 − λx1 = y1 −y1 − y2 x1 − x2× x1 =x1y2 − x2y1 x1 − x2(3.7) Tiếp theo thay y ở (3.2) vào phương trình (2.2) ta có: (λx + µ)2 + (a1x + a3)(λx + µ) = x3 + a2x2 + a4x + a6 (3.8) Từ đó dẫn đến phương trình r(x) = 0 với: r(x) = x3 + (a2 − λ2 − a1λ)x2 + (a4 − 2λµ − a3λ − a1µ)x + a6 − µ2 − a3µ (3.9) 30 Tạp chí Epsilon, Số 09, 06/2016 Biết rằng r(x) có 3 nghiệm phân biệt nên có thể viết: r(x) = (x − x1)(x − x2)(x − x3) = (x2 − (x1 + x2)x + x1x2)(x − x3) = x3 − (x1 + x2)x2 + x1x2x − x3x2 + (x1 + x2)x3x − x1x2x3 = x3 − (x1 + x2 + x3)x2 + (x1x2 + x1x3 + x2x3)x − x1x2x3 (3.10) Đồng nhất các hệ số x2của r(x) ở 2 phương trình (3.9) và (3.10) ta có: x1 + x2 + x3 = −(a2 − λ2 − a1λ) từ đây có thể tính được x3 theo công thức sau: x3 = λ2 + a1λ − a2 − x1 − x2 (3.11) Đến đây cần phải tính tiếp giá trị y3, lúc này x3 đã tính xong nên có thể coi là hằng số, có thể viết lại (2.2) thành dạng sau: y2 + (a1x3 + a3)y − (x33 + a2x23 + a4x3 + a6) = 0 (3.12) Phương trình bậc 2 này có 2 nghiệm là: y3, y03 =−(a1x3 + a3) ±√∆ 2 × 1(3.13) Cộng 2 nghiệm này ta sẽ có: y03 + y3 = −a1x3 − a3, mặt khác do y03 nằm trên đường thẳng P1, P2 nên y03 = λx3 + µ. Từ đây có thể tính được y3 theo công thức 5: y3 = −λx3 − µ − a1x3 − a3 (3.14) Thay µ từ (3.7) ta có thể tính y3 dưới dạng sau: y3 = λ(x1 − x3) − y1 − a1x3 − a3 (3.15) 3.2. Trường hợp 2 điểm trùng nhau P1 = P2 Khi này x1 = x2 và y1 = y2 do đó công thức tính λ ở (3.7) không sử dụng được vì xuất hiện phép chia số 0. Trong trường hợp này λ chính là hệ số góc của đường thẳng tiếp tuyến đường cong E tại P1 hay P2. Hệ số góc của tiếp tuyến của E chính là đạo hàm dy dx , sử dụng các quy tắc lấy đạo hàm của tích, đạo hàm của hàm số hợp và lấy đạo hàm 2 vế của phương trình (2.2) theo dx ta có: d(y2 + a1xy + a3y) dx =d(x3 + a2x2 + a4x + a6) dx d(y2) dx +d(a1xy) dx +d(a3y) dx = 3x2 + 2a2x + a4 d(y2) dy×dydx + a1(d(xy) dx ) + a3dydx = 3x2 + 2a2x + a4 2ydydx + a1(y + xdydx) + a3dydx = 3x2 + 2a2x + a4 (2y + a1x + a3)dydx = 3x2 + 2a2x + a4 − a1y dx =3x2 + 2a2x + a4 − a1y dy 2y + a1x + a3 5Ghi chú: Các tác giả H. Cohen và G. Frey trong cuốn “Handbook of Elliptic and Hyperelliptic Curve Cryptog raphy” [7] trình bày diễn giải về cách tính y3 dựa trên sự đối xứng qua trục x là không chính xác và thiếu rõ ràng đối với đường cong Elliptic dạng tổng quát, cách giải thích của người viết bằng cách giải phương trình bậc 2 ở đây có thể coi là đúng đắn và rõ ràng hơn. 31 Tạp chí Epsilon, Số 09, 06/2016 Như vậy với điểm P1(x1, y1) ta có: λ =3x21 + 2a2x1 + a4 − a1y1 2y1 + a1x1 + a3(3.16) Trong tất cả các trường hợp điểm P3 là tổng của 2 điểm P1, P2 sẽ là điểm có tọa độ là: P3(x3, y3) = (λ2 + a1λ − a2 − x1 − x2, λ(x1 − x3) − y1 − a1x3 − a3) (3.17) Với đường cong E dạng (2.1), khi đó a1 = a3 = a2 = 0 và P3 sẽ được tính theo công thức: P3(x3, y3) = (λ2 − x1 − x2, λ(x1 − x3) − y1) (3.18) Trong trường hợp P1 = P2, (3.16) sẽ được biến đổi thành: λ =3x21 + a4 2y1(3.19) 4. Nhân vô hướng các điểm trên đường cong Elliptic Với n ∈ N \ {0} định nghĩa phép nhân vô hướng của điểm P nằm trên đường cong E là phép cộng n lần chính bản thân điểm P: P 7→ nP = P + P + · · · + P | {z } n lần = Q Để tối ưu phép nhân vô hướng, có thể sử dụng phương pháp Nhân đôi-và-cộng, đầu tiên biểu diễn số n dưới dạng: n = n0 + 2n1 + 22n2 + · · · + 2mnm với [n0 . . . nm] ∈ {0, 1}, sau đó áp dụng thuật toán: Algorithm 1 Phương pháp Nhân đôi-và-cộng 1: Q ← 0 2: for i = 0 to m do 3: if ni = 1 then 4: Q ← CộngĐiểm(Q,P) 5: end if 6: P ← NhânĐôi(P) 7: end for 8: return Q Ngoài phương pháp Nhân đôi-và-cộng, có thể sử dụng phương pháp Trượt-cửa-sổ. Các phương pháp này cho phép nhân vô hướng một cách tối ưu. Lưu ý của người viết: • Không tồn tại phép nhân 2 điểm trên đường cong E, có nghĩa là không tồn tại P × Q với P, Q ∈ E. • Không tồn tại thuật toán chia vô hướng Q : n. Biết rằng Q = nP, bài toán tìm số n là bài toán Logarithm rời rạc sẽ được đề cập tới ở chương sau. Đây là bài toán khó, thông thường phải thử lần lượt n = 1, 2, . . . , n − 1 phép cộng điểm P, cho đến khi tổng bằng Q, tuy nhiên có một số thuật toán tối ưu hơn để tìm n nhưng vẫn không thể giải được bài toán này trong thời gian đa thức vì thế dựa vào độ khó này có thể xây dựng ra hệ mật đường cong Elliptic với các giao thức cho mã hóa, xác thực và trao đổi khóa. 32 Tạp chí Epsilon, Số 09, 06/2016 Hình 6.2: Ví dụ về tính chất kết hợp trên đường cong Elliptic 5. Nhóm (+) của các điểm trên đường cong Elliptic Xét đường cong Elliptic E được định nghĩa bởi phương trình y2 = x3 + Ax + B. Xét 3 điểm nằm trên đường cong E là P1, P2, P3 lần lượt có các tọa độ là (x1, y1),(x2, y2),(x3, y3). 33 Tạp chí Epsilon, Số 09, 06/2016 Để các điểm trên đường cong Elliptic tạo thành nhóm (+), “điểm vô cùng” (∞) sẽ được thêm vào đường cong, kí hiệu là O, điểm này sẽ nằm ở trên cùng và dưới cùng của trục y. Một trong những thuộc tính quan trọng nhất của đường cong Elliptic là tồn tại nhóm các điểm với phép cộng nằm trên đường cong. Định lý 5.1. Phép cộng với các điểm P, P1, P2, P3 trên đường cong E thỏa mãn các tính chất của nhóm: 1. (Giao hoán): P1 + P2 = P2 + P1; 2. (Điểm đơn vị): P + ∞ = P; 3. (Điểm nghịch đảo): Tồn tại P0của P sao cho P + P0 = ∞; 4. (Kết hợp): (P1 + P2) + P3 = P1 + (P2 + P3). Chứng minh. (1. Tính chất giao hoán) của phép cộng 2 điểm P1, P2 là hiển nhiên từ công thức tính tọa độ của điểm tổng, các giá trị có giao hoán thì giá trị tính bởi công thức này cũng không thay đổi, hoặc về mặt hình học đường thẳng đi qua P1, P2 dù có xuất phát từ P1 hay P2 thì đều như nhau và cùng cắt đường cong E tại một điểm chung duy nhất. (2. Điểm đơn vị) không cần phải chứng minh vì nó xuất phát từ định nghĩa. Có thể lý giải rõ hơn về điểm (∞) và cách định nghĩa phép cộng trên E (theo quan điểm của người viết) như sau: Khi đường cong E không suy biến, nó sẽ cắt một đường thẳng được định nghĩa bởi phương trình (3.2) ở 3 điểm, thực vậy theo các phép biến đổi ở mục 3.1 (trang 28) phương trình (3.9) r(x) sẽ có 3 nghiệm phân biệt. Mặt khác E đối xứng qua trục x do phương trình (2.1) có thành phần y2 nên luôn tồn tại hai giá trị y, −y thỏa mãn (2.1), cũng do tính đối xứng này nên đường cong E sẽ cắt các đường thẳng song song với với trục y ở 2 điểm, vì nếu cắt thêm 1 điểm nữa thì sẽ phải cắt thành 4 điểm do tính đối xứng, điều này là mẫu thuẫn vì phương trình bậc 3 chỉ có tối đa 3 nghiệm. Ở trường hợp này nếu cộng 2 điểm nằm trên đường song song trục y sẽ không tìm được điểm thứ 3 do vậy P1 + P2 sẽ không tồn tại. Chính vì để nhóm các điểm trên E có tính đóng bắt buộc chúng ta phải định nghĩa thêm điểm ∞ coi như là điểm thứ 3 nằm trên đường cong E, và nó sẽ nằm ở vô cực ở 2 đầu trục y. Tiếp theo, có thể lý giải (của người viết6) về phép định nghĩa phép cộng 2 điểm trên E như sau. Để thỏa mãn tính chất tồn tại điểm đơn vị theo định nghĩa về nhóm G với mọi giá trị a ∈ G tồn tại e ∈ G sao cho a • e = e • a = a (5.1) Xét điểm P trên đường cong E, khi đó cần tính P + ∞, dễ thấy điểm này chắc chắn phải nằm trên cùng đường thẳng song song trục y vì nếu không sẽ cắt E ở 2 điểm nữa cùng với ∞ tạo thành 4 điểm và điều này là phi lý. Nếu đã nằm trên đường song song y thì nó sẽ cắt E ở điểm đối xứng qua trục x, nếu coi điểm cắt này là tổng P + ∞ thì như vậy sẽ tồn tại P + ∞ = P0 và điều kiện (5.1) sẽ không được thỏa mãn. Vì vậy sẽ phải định nghĩa điểm tổng không phải là giao 6Khi bắt đầu nghiên cứu về đường cong Elliptic, luôn có 2 câu hỏi mà người viết không thể tìm thấy trong nhiều tài liệu kể cả những cuốn kinh điển về Elliptic: 1. Tại sao phải chọn ∞ làm điểm trung hòa. 2. Tại sao P1 + P2 = P3 mà P3 không nằm trên đường thẳng đi qua P1, P2 mà phải là điểm đối xứng của giao điểm qua trục x. Trong chuyên đề này cả 2 câu hỏi đều đã được người viết lý giải một cách rõ ràng và đầy đủ. 34 Tạp chí Epsilon, Số 09, 06/2016 trực tiếp với E mà phải lấy là điểm đối xứng để đối xứng của điểm đối xứng sẽ quay lại chính P và ta có P + ∞ = P. (3.Điểm nghịch đảo) Cũng từ nhận xét rằng luôn tồn tại 2 điểm P, P0 nằm trên cùng đường thằng song song với trục y và sẽ cắt đường cong E ở điểm ∞, coi 2 điểm này là nghịch đảo của nhau và sẽ luôn có: P + P0 = ∞. (4. Tính chất kết hợp) Chứng minh (P1 + P2) + P3 = P1 + (P2 + P3) khác hẳn với 3 điều kiện khác về nhóm, và nó đặc biệt phức tạp. Có 2 cách chứng minh điều này là dùng phương pháp hình học hoặc đại số. Có thể tham khảo chứng minh bằng hình học qua các tài liệu [3], [17] và [4], tuy nhiên chứng minh bằng phương pháp hình học tương đối khó hiểu với một số định lý trong không gian xạ ảnh. Dưới đây là một số điểm được sử dụng để chứng minh bằng phương pháp đại số. Trước tiên tính các điểm tổng sau: P12 = P1 + P2 P32 = P3 + P2 P123 = (P1 + P2) + P3 = P12 + P3 P321 = (P3 + P2) + P1 = P32 + P1 Từ các công thức tính giá trị tọa độ (x, y) của điểm tổng, có thể dễ dàng biến đổi tính điểm: λ12 =(y2 − y1) (x2 − x1)(5.2) λ32 =(y2 − y3) (x2 − x3)(5.3) x12 = λ212 − x1 − x2 =(y2 − y1)2 (x2 − x1)2− x1 − x2 (5.4) y12 = λ12(x1 − x12) − y1 = −λ312 + (2x1 + x2)λ12 − y1 (5.5) = −(y2 − y1)3 (x2 − x1)3+(2x1 + x2)(y2 − y1) x2 − x1− y1 x32 = λ232 − x3 − x2 =(y2 − y3)2 (x2 − x3)2− x3 − x2 (5.6) y32 = λ32(x3 − x32) − y3 = −λ332 + (2x3 + x2)λ32 − y3 (5.7) = −(y2 − y3)3 (x2 − x3)3+(2x3 + x2)(y2 − y3) x2 − x3− y3 λ123 =(y12 − y3) (x12 − x3)(5.8) λ321 =(y32 − y1) (x32 − x1)(5.9) Cuối cùng cách tính tọa độ của các điểm P123, P321 theo tọa độ 3 điểm P1(x1, y1), P2(x2, y2), P3(x3, y3) 35 Tạp chí Epsilon, Số 09, 06/2016 được biểu diễn dưới các dạng công thức từ (5.10) đến (5.13). −(y2 − y1)3 (x2 − x1)3+(2x1 + x2)(y2 − y1) x2 − x1− y1 − y3 2 x123 = (y2 − y1)2 (x2 − x1)2− x1 − x2 − x3 2 −(y2 − y1)2 (x2 − x1)2+ x1 + x2 − x3 (5.10) y123 = −(y2 − y1)3 (x2 − x1)3+(2x1 + x2)(y2 − y1) x2 − x1− y1 − y3 3 (y2 − y1)2 (x2 − x1)2− x1 − x2 − x3 3(5.11) + 2(y2 − y1)2 (x2 − x1)2− 2x1 − 2x2 + x3 −(y2 − y1)3 (x2 − x1)3+(2x1 + x2)(y2 − y1) x2 − x1− y1 − y3 (y2 − y1)2 (x2 − x1)2− x1 − x2 − x3 +(y2 − y1)3 (x2 − x1)3−(2x1 + x2)(y2 − y1) x2 − x1+ y1 −(y2 − y3)3 (x2 − x3)3+(2x3 + x2)(y2 − y3) x2 − x3− y3 − y1 2 x321 = (y2 − y3)2 (x2 − x3)2− x3 − x2 − x1 2 −(y2 − y3)2 (x2 − x3)2+ x3 + x2 − x1 −(y2 − y3)3 (x2 − x3)3+(2x3 + x2)(y2 − y3) x2 − x3− y3 − y1 3 (5.12) y321 = (y2 − y3)2 (x2 − x3)2− x3 − x2 − x1 3(5.13) + 2(y2 − y3)2 (x2 − x3)2− 2x3 − 2x2 + x1 −(y2 − y3)3 (x2 − x3)3+(2x3 + x2)(y2 − y3) x2 − x3− y3 − y1 (y2 − y3)2 (x2 − x3)2− x3 − x2 − x1 +(y2 − y3)3 (x2 − x3)3−(2x3 + x2)(y2 − y3) x2 − x3+ y3 Cần phải chứng minh rằng P123 = P321 điều này có nghĩa cần phải chứng minh: dx = x123 − x321 = 0 (5.14) dy = y123 − y321 = 0 (5.15) Triển khai vế trái của (5.14), sẽ được một phân số mà tử số và mẫu số bao gồm tổng cộng 1446 thành phần, tương tự vế trái của (5.15) có tất cả 10081 thành phần có dạng nkxi11 xi22 xi33yj1 1yj2 2yj3 3, trong đó số mũ i1, i2, i3, j1, j2, j3 nằm trong khoảng [0..12] và nk là hệ số của thành phần biểu 36 Tạp chí Epsilon, Số 09, 06/2016 thức trên. Có thể kiểm tra tính đúng đắn của (5.14) và (5.15) bằng phần mềm Maple với các ràng buộc sau: y21 = x31 + Ax1 + B (5.16) y22 = x32 + Ax2 + B (5.17) y23 = x33 + Ax3 + B (5.18) 6. Đường cong Elliptic trên trường hữu hạn Fq 6.1. Trường hữu hạn Fq Các ứng dụng về mật mã của đường cong Elliptic đa số chỉ sử dụng các đường cong trên trường hữu hạn. Xét Fq là một trường hữu hạn (hữu hạn số phần tử số nguyên dương): Fq = {0, 1, 2 . . . , q − 1} q là một số nguyên tố hoặc có dạng q = pm với p là một số nguyên tố và m là một số nguyên dương. Khi này p được gọi là đặc số char(q) = p và m là bậc mở rộng của Fq. Trong thực tế và đặc biệt trong các thiết bị phần cứng [18], người ta thường sử dụng trường hữu hạn F2m. Khi đó phép cộng trong trường này đơn giản chỉ là phép toán XOR (Exclusive OR). Nhiều tài liệu cho thấy làm việc với F2m hiệu quả hơn 40% so với làm việc với trường Fq. Nhóm thực hiện Đề tài cấp Nhà nước KC01.18 do người viết làm hủ nhiệm đề tài đã cài đặt toàn bộ các phép toán về đường cong Elliptic trên trường F2m cho Chip Spartan 6 của Xilinx cho bài toán xác thực và trao đổi khóa phiên trong thiết bị VPN IPSec. Trường F2m thường được biểu diễn dưới dạng tổ hợp tuyến tính của các vector gồm m phần tử {α0, α1, . . . , αm−1}, mọi phần tử α ∈ F2m đều có thể được biển diễn dưới dạng: α = a0α0 + a0α0 + . . . + am−1αm−1, ai ∈ {0, 1} Có nhiều phương pháp để xây dựng cơ sở của F2m: đa thức cơ sở và cơ sở chuẩn tắc. Các thuật toán để thực hiện các phép toán trên EC có thể tìm thấy trong [19]. 6.1.1. Đa thức cơ sở Xét đa thức f(x) = xm + fm−1xm−1 + . . . + f2x2 + f1x + f0 (với fi ∈ F2, i = 0, . . . , m − 1) là một đa thức bất khả quy bậc m trên trường F2, nghĩa là không thể phân tích f(x) thành các đa thức thừa số khác có bậc nhỏ hơn m. f(x) gọi là đa thức rút gọn. Trường hữu hạn F2m sẽ là tập tất cả các đa thức trên F2 có bậc nhỏ hơn hoặc bằng m. F2m = {am−1xm−1 + . . . + a2x2 + a1x + a0 : ai ∈ {0, 1}} Các phần tử (am−1xm−1 + . . . + a2x2 + a1x + a0) thường được biểu diễn dưới dạng chuỗi bit (am−1 . . . a1a0) có độ dài là m. Các phép toán trong trường F2m: • Phép cộng: (cm−1 . . . c1c0) = (am−1 . . . a1a0) + (bm−1 . . . b1b0), ci = ai ⊕ bi. 37 Tạp chí Epsilon, Số 09, 06/2016 • Phép nhân: (rm−1 . . . r1r0) = (am−1 . . . a1a0).(bm−1 . . . b1b0) Trong đó (rm−1xm−1 + . . . + r1x + r0) = (am−1xm−1 + . . . + a1x + a0) × (bm−1xm−1 + . . . + b1x + b0) mod f(x). Đa thức rút gọn f(x) thường có dạng sau: • Trinomial basis (TPB): f(x) = xm + xk + 1, 1 6 k 6 m − 1. • Pentanomial basis (PPB): f(x) = xm + xk3 + xk2 + xk1 + 1, 1 6 k1 < k2 < k3 6 m − 1. 6.1.2. Cơ sở chuẩn tắc F2m sử dụng cơ sở có dạng {β, β22, . . . , β2m−1} với β ∈ F2m, khi đó mỗi phần tử a ∈ F2m đều có dạng a =Pm−1 i=0 aiβ2i, ai ∈ {0, 1} và cũng được biểu diễn dưới dạng chuỗi bit (a0a1 . . . am−1) có độ dài là m. Với cơ sở này phép bình phương sẽ thực hiện rất đơn giản chỉ bằng cách quay bit. Các phép toán trong trường F2m: • Phép cộng: (c0c1 . . . cm−1) = (a0a1 . . . am−1) + (b0b1 . . . bm−1), ci = (ai + bi) mod 2. • Phép bình phương: a2 = • Phép nhân: mX−1 i=0 aiβ2i !2 = mX−1 i=0 aiβ2i+1= mX−1 i=0 ai−1β2i= (am−1a0a1 . . . am−2) Xét p = Tm + 1 và u ∈ Fp là phần tử bậc T, định nghĩa chuỗi F(1), F(2), . . . , F(p − 1) ta sẽ có: F(2iuj mod p) = i, 0 6 i 6 m − 1, 0 6 j 6 T − 1. (c0c1 . . . cm−1) = (a0a1 . . . am−1).(b0b1 . . . bm−1) cl =  Pp=2 k=1 aF(k+1)+lbF(p−k)+l, nếu T chẵn. Pm/2 k=1(ak+l−1bm/2+k+l−1 + am/2+k+l−1bk+l−1)  +Pp=2 k=1 aF(k+1)+laF(k+1)+l, nếu T lẻ. 6.2. Tổng số điểm của đường cong Elliptic trên trường hữu hạn Fq E là đường cong Elliptic trên trường Fq, bởi vì cặp (x, y) với x, y ∈ Fq là hữu hạn do đó nhóm E(Fq) cũng sẽ là nhóm hữu hạn. Các giá trị x, y là các số nguyên, dễ dàng nhận thấy không phải với mọi giá trị x đều tìm được giá trị nguyên y bởi vì không phải bao giờ √y cũng là một số nguyên dương. Câu hỏi đặt ra là số điểm của của đường cong Elliptic trên trường Fq là bao nhiêu? Xác định số điểm trên đường cong E nhằm xác định không gian khóa của hệ mật. Sau đây là phần trình bày về việc tính tổng số điểm của đường cong Elliptic trên trường hữu hạn Fq. 38 Tạp chí Epsilon, Số 09, 06/2016 Bổ đề 1. M và N là hai ma trận 2×2. M = a, b ta có: m11 m12 m21 m22 , N = n11 n12 n21 n22 , với các số nguyên det(aM + bN) = a2det(M) + b2det(N) + ab(det(M + N) − det(M) − det(N)) (6.1) Chứng minh. Theo định nghĩa về định thức ta có: det(M) = m11m22 − m21m12, det(N) = n11n22 − n21n12 det(aM + bN) = det am11 + bn11 am12 + bn12 am21 + bn21 am22 + bn22 = (am11 + bn11)(am22 + bn22) − (am21 + bn21)(am12 + bn12) = a2(m11m22 − m21m12) + b2(n11n22 − n21n12)+ + ab(m11n22 + n11m22 − m21n12 − n21m12) = a2det(M) + b2det(N) + ab(m11n22 + n11m22 − m21n12 − n21m12) (6.2) Khi a = b = 1 áp dụng công thức ở trên ta có: det(M + N) = det(M) + det(N) + (m11n22 + n11m22 − m21n12 − n21m12) (6.3) Nhân cả 2 vế (6.3) với ab sau đó trừ vào (6.2) sẽ được kết quả (6.1) là điều cần phải chứng minh. Định nghĩa 6.1. Điểm n-xoắn (Torsion): Cho đường cong Elliptic E được định nghĩa trên trường K, cho n là số nguyên dương, tập các điểm Torsion E[n] là tập các điểm trên đường cong có tính chất như sau: K là đóng đại số của K. E[n] = P ∈ E(K) | nP = ∞ (6.4) Định nghĩa 6.2. Divisor: Cho đường cong Elliptic E được định nghĩa trên trường K, với mỗi điểm P ∈ E(K) định nghĩa một ký hiệu hình thức (formal symbol) [P], khi đó divisor D sẽ là: D =X j aj[Pj], aj ∈ Z f là một hàm trên E mà khác 0, khi đó divisor của f sẽ là: div(f) = X P ∈E(K) ordP (f)[P] ∈ Div(E) f = urPg, với r ∈ Z và g(P) 6= 0, ∞, u(P) = 0, định nghĩa bậc của f tại P là: ordP (f) = r. Định nghĩa 6.3. Cặp Weil: là một ánh xạ từ 2 điểm trong nhóm các điểm Torsion thành giá trị bậc thứ n của đơn vị: en : E[n] × E[n] → µn, µn = {x ∈ K | xn = 1} 39 Tạp chí Epsilon, Số 09, 06/2016 Bổ đề 2. Với mọi tự đồng cấu bất khả tách α trên E, và với mọi S, S1, S2, T, T1, T2 ∈ E[n] ta có: en(α(S), α(T)) = en(S, T)deg(α) en(T, T) = 1 en(S1 + S2, T) = en(S1, T)en(S2, T) en(S, T1 + T2) = en(S, T1)en(S, T2) Chứng minh có thể xem trong [3]. Giả thiết {T1, T2} là cơ sở của E[n], mỗi phần tử trong E[n] đều có thể biểu diễn dưới dạng tổ hợp tuyến tính m1T1 + m2T2. α là một tự đồng cấu trong E[n], n là một số nguyên không chia hết bởi char(K). Tồn tại các số a, b, c, d ∈ Z sao cho: α(T1) = aT1 + cT2, α(T2) = bT1 + dT2 Do đó mỗi tự đồng cấu α đều có thể được biểu diễn bởi ma trận 2 × 2: αn = a b c d Bổ đề 3. α là một tự đồng cấu trong E[n], n là một số nguyên không chia hết bởi char(K) khi đó det(αn) ≡ deg(α) mod n. Chứng minh. Đặt ζ = en(T1, T2), theo bổ đề 2 ta có: ζdeg(α) = en(α(T1), α(T2)) = en(aT1 + cT2, bT1 + dT2) = en(T1, T1)aben(T1, T2)aden(T2, T1)cben(T2, T2)cd = ζad−bc = ζdet(αn) Nếu α, β là 2 tự đồng cấu trên E, và a, b là các số nguyên thì tự đồng cấu aα + bβ được định nghĩa như sau: (aα + bβ)(P) = aα(P) + bβ(P) Bổ đề 4. deg(aα + bβ) = a2 deg α + b2 deg β + ab(deg(α + β) − deg α − deg β) Chứng minh. Biểu diễn các tự đồng cấu α, β bằng các ma trận αn, βn (với một số cơ sở trong E[n]), theo đó aα + bβ sẽ được biểu diễn bằng aαn + bβn. Áp dụng công thức (6.1) ta có: det(aαn + bβn) = a2det(αn) + b2det(βn) + ab(det(αn + βn) − det(αn) − det(βn)) Theo bổ đề 3 chúng ta sẽ có: deg(aα + bβ) = a2deg(α) + b2deg(β) + ab(deg(α + β) − deg(α) − deg(β)) 40 Tạp chí Epsilon, Số 09, 06/2016 Định lý 6.4. (Hasse) Nếu E là đường cong Elliptic trên trường Fq, và #E(Fq) là tổng số điểm trên đường cong đó thì: q + 1 − 2√q 6 #E(Fq) 6 q + 1 + 2√q. (6.5) Chứng minh. Trước tiên xét ánh xạ Probenius được định nghĩa như sau: φq : Fq −→ Fq, x 7→ xq Có thể viết một cách khác: φq(x, y) = (xq, yq), φq(∞) = ∞ Khi thay các giá trị xq, yq vào phương trình (2.1) dễ thấy (x, y) cũng nằm trên đường cong E. Ánh xạ φq là một tự đồng cấu và có thể biểu diễn bằng hàm đa thức hữu tỷ có bậc là q. Đạo hàm của xqlà qxq−1sẽ bằng 0 bởi vì q = 0 trong trường Fq. Do đạo hàm bằng 0 nên φq là khả tách (separable). Bởi vì φq là tự đồng cấu trong E do đó φ2q = φq ◦ φq cũng là tự đồng cấu và φnqcũng là tự đồng cấu trong E. Phép nhân với −1 cũng là tự đồng cấu do đó tổng φnq − 1 là đồng cấu trong E. φq là khả tách (separable) nhưng φq − 1 sẽ là bất khả tách do đó bậc của nó sẽ bằng số phần tử của hạch φq − 1 có nghĩa là số điểm trên đường cong E sẽ là: #E(Fq) = deg(φq − 1) Với các số nguyên r, s, áp dụng bổ đề 4 ta có: deg(rφq − s) = r2deg(φq) + s2deg(−1) + rs(deg(φq − 1) − deg(φq) − deg(−1)) Bởi vì deg(−1) = 1 và deg(φq) = q nên: deg(rφq − s) = r2q + s2 + rs(deg(φq − 1) − q − 1)) Đặt a = −(deg(φq − 1) − q − 1) = q + 1 − #E(Fq), bởi vì deg(rφq − s) > 0 suy ra r2q + s2 + rsa > 0 hay với mọi r, s ta có: q r s 2− a rs + 1 > 0 Do đó ∆ = a2 − 4q 6 0 hay là |a| 6 2√q cũng có nghĩa là |q + 1 − #E(Fq)| 6 2√q và đó là điều phải chứng minh. Tham khảo thêm về cách tính số điểm trên đường cong E có thể xem trong [20]. Tài liệu tham khảo [1] J. W. Bos, J. A. Halderman, N. Heninger, J. Moore, M. Naehrig, and E. Wustrow, “Elliptic Curve Cryptography in Practice,” Financial Cryptography and Data Security, vol. 8437, pp. 157–175, 2014. 41 Tạp chí Epsilon, Số 09, 06/2016 [2] J. H. Silverman and J. T. Tate, Rational Points on Elliptic Curves - Second Edition. Springer, 2015. [3] L. C. Washington, Elliptic Curves Number Theory and Cryptography, Second Edition. CRC Press, 2008. [4] J. W. S. Cassels, Lectures on Elliptic Curves. University of Cambridge, 1991. [5] S. Lang, Elliptic Curves Diophantine Analysis. Springer, 1978. [6] C. Kenig, A. Ranicki, and M. Rockner, Elliptic Curves A Computational Approach. Walter de Gruyter GmbH & Co., 2003. [7] H. Cohen and G. Frey, Handbook of Elliptic and Hyperelliptic Curve Cryptography. Chapman Hall/CRC, 2006. [8] J. H. Silverman, The Arithmetic of Elliptic Curves. Springer, 2009. [9] L. Berger, G. Bockle, L. D. M. Dimitrov, T. Dokchitser, and J. Voight, Elliptic curves, Hilbert modular forms and Galois deformations. Birkhauser, 2013. [10] I. F. Blake, G. Seroussi, and N. P. Smart, Advances in Elliptic Curve Cryptography. Cambridge University Press, 2005. [11] I. Connell, Elliptic Curve Handbook. McGill University, 1999. [12] T. H. Otway, Elliptic Hyperbolic Partial Differential Equations. Springer, 2015. [13] Dang Minh Tuan, “Che tao thiet bi VPN IPSec bang phan cung dau tien o Vietnam,” Tap chi CNTT & TT, no. 2, pp. 41–45, 2014. [14] H. Lenstra., “Factoring Integers with Elliptic Curves,” The Annals of Matematics, vol. 126, no. 3, pp. 649–673, 1987. [15] V. S. Miller, “Use of elliptic curves in cryptography,” CRYPTO ’85, pp. 417–428, 1985. [16] N. Koblitz, “Elliptic curve cryptosystem,” Math.Comp, vol. 48, no. 16, pp. 203–209, 1987. [17] A. Enge, Elliptic Curves and Their Applications to Cryptography. Kluwer Academic Publishers, 2001. [18] D. Hankerson, J. L. Hernandez, and A. Menezes, “Software Implementation of Elliptic Curve Cryptography over Binary Fields,” CHES2000, vol. 1965, pp. 243–267, 2000. [19] D. Hankerson, A. Menezes, and S. Vanstone, Guide to Elliptic Curve Cryptography. Springer-Verlag, 2004. [20] R. Schoof, “Elliptic Curves Over Finite Fields and the Computation of Square Roots,” Matematics of Computation, pp. 483–495, 1985. [21] I. Blake, G. Seroussi, and N. Smart, Elliptic Curves in Cryptography. Cambridge Univer sity Press, 1999. [22] H. Cohen, A Course in Computational Algebraic Number Theory. Springer-Verlag, 1993. 42 Tạp chí Epsilon, Số 09, 06/2016 [23] J. M. Pollard, “Monte Carlo Methods for Index Computations (mod p),” Mathematics of Computation, vol. 32, no. 143, pp. 918–924, 1978. [24] S. C. Pohlig and M. E. Hellman, “An Improved Algorithm for Computing Logarithms over GF(p) and its Cryptographic Significance,” IEEE Transactions on Information Theory, vol. 24, pp. 106–110, 1978. [25] A. J. Menezes, T. Okamoto, and S. A. Vanstone, “Reducing elliptic curve logarithms to logarithms in a finite field,” IEEE Trans. Inform. Theory, vol. 39, no. 5, pp. 1639–1646, 1993. [26] C. Research, Standards For Efficient Cryptography, SEC 1: Elliptic Curve Cryptography. Certicom Corp, 2000. [27] L. Gao, S. Shrivastava, and G. E. Sobelman, “Elliptic Curve Scalar Multiplier Design Using FPGAs,” CHES’99, vol. 1717, pp. 257–268, 1999. [28] L. Laurie, M. Alfred, Q. Minghua, S. Jerry, and V. Scott, “An Efficient Protocol for Authenticated Key Agreement,” Designs Codes and Cryptography, vol. 28, no. 2, 1998. [29] D. Johnson, A. Menezes, and S. Vanstone, “The Elliptic Curve Digital Signature Algorithm (ECDSA),” 2001. [30] T. E. Gamal, “A Public Key Cryptosystem and a Signature Scheme Based on Discrete Logarithms,” CRYPTO ’84, vol. 196, pp. 10–18, 1985. [31] NIST, Digital Signature Standard (DSS) FIPS 186-4. National Institute of Standards and Technology, 2013. [32] J. Massey and J. Omura, “Method and apparatus for maintaining the privacy of digital messages conveyed by public transmission,” Jan. 28 1986, US Patent 4,567,600. [Online]. Available: https://www.google.com/patents/US4567600 43 Tạp chí Epsilon, Số 09, 06/2016 44 NGHỊCH LÝ TRAI GÁI Đặng Nguyễn Đức Tiến (Đại học Trento, Italia) LỜI GIỚI THIỆU Tiếp nối các chuyên mục Toán học Giải trí ở những số Epsilon trước, số này chúng tôi giới thiệu với độc giả một nghịch lý nổi tiếng trong xác suất: Nghịch lý trai gái. Bài toán này, lại một lần nữa như rất nhiều bài toán đố khác trong chuyên mục Toán học Giải trí, xuất phát từ cây đại thụ Martin Gardner1 với tên gọi là bài toán hai người con (Two Children Problem) đăng ở tờ Scientific American vào năm 1954: "Một người có hai con, biết rằng có ít nhất một con trai, hỏi khả năng cả hai đều là con trai là bao nhiêu?" 1. Nghịch lý trai gái và bài toán ngày thứ ba Chúng ta hãy bắt đầu với việc phân tích thử lời giải cho bài toán về hai người con của Martin. Để đơn giản, hãy cùng thống nhất với giả thiết là giới tính của mỗi người con là độc lập (nghĩa là giới tính người em không phụ thuộc vào giới tính của anh hoặc chị người đó) và khả năng mỗi người con là nam hay nữ là bằng nhau. Trực quan và dễ thấy, vì ít nhất một người là trai, nên người còn lại sẽ hoặc trai hoặc gái với khả năng bằng nhau (theo giả thiết). Khả năng cả 2 đều là con trai do vậy là 1/2. Vậy đâu có gì là mâu thuẫn hay nghịch lý! Nhưng dừng lại một chút, chúng ta hãy thử phân tích các khả năng có thể có của 2 người con khi chưa biết giới tính của họ. Có thể có 4 trường hợp ở đây theo thứ tự người con thứ 1 và người con thứ 2 là: nam - nam, nam - nữ, nữ - nam và nữ - nữ, khả năng xảy ra như nhau. Và vì ta biết ít nhất một trong 2 người là nam, nên trường hợp cuối nữ - nữ bị loại ra, chỉ còn có 3 trường hợp đầu và trong số này chỉ có trường hợp nam - nam là ứng với yêu cầu của câu hỏi. Vì vậy, khả năng cả 2 đều là con trai là 1/3. Nghịch lý vì lý luận ở trên chỉ ra khả năng là 1/2! Nghịch lý này thu hút được rất nhiều người, đặc biệt là có thời điểm người ta đã chia ra làm hai “trường phái" đối lập hẳn nhau là trường phái “nhị phân" và trường phái "tam phân", ứng với 2 lý luận như đã giải thích ở trên. Và sau hơn 60 năm từ khi ra đời cho đến nay, sự mâu thuẫn giữa hai trường phái, hay có thể nhìn nhận như mâu thuẫn giữa trực quan và lý luận đối với bài toán này vẫn còn xảy ra với rất nhiều người và câu trả lời đâu là đáp án đúng thật sự không phải là đơn giản, vì thoạt nhìn, ai cũng ... có lý. Trước khi đi vào phân tích đâu là trường phái "chiến thắng" ở nghịch lý trai gái, chúng tôi giới thiệu tiếp với độc giả một phiên bản lạ của bài toán này, đề xuất bởi Gary Foshee vào năm 2010. Bài toán như sau: 1Bạn đọc có thể xem lại ở Epsilon số 3 về cuộc đời của nhà toán học huyền thoại này. 45 Tạp chí Epsilon, Số 09, 06/2016 "Một gia đình có hai con, nếu như biết rằng ít nhất một có một đứa là con trai và sinh vào ngày thứ ba thì khả năng để cả hai đều là con trai là bao nhiêu?" Hẳn nhiều bạn đọc sẽ hỏi: "Thêm vào ngày thứ ba thì liên quan gì?" Và câu trả lời hẳn sẽ làm ngạc nhiên cho bạn vì xác suất để cả hai đều là con trai không phải 1/2 hay 1/3 mà là 13/27. 2. Đi tìm lời giải Trước tiên, chúng tôi hệ thống lại hai bài toán như sau: Một gia đình có hai người còn, hỏi khả năng cả 2 đều là con trai là bao nhiêu, biết rằng: 1. Xác suất của nam và nữ là bằng nhau và bằng 1/2. 2. Giới tính của hai người không phụ thuộc vào nhau. 3. Cho biết trước một điều kiện K. Và tổng hợp lại các trường hợp khác nhau vào bảng sau: Trường hợp Điều kiện K TH1 Không có điều kiện TH2 Người con đầu là con trai TH3 Ít nhất một trong hai là con trai TH4 Ít nhất một trong hai là con trai và sinh vào ngày thứ ba Bây giờ, chúng ta hãy cùng xét trường hợp đơn giản trước TH1, không gian mẫu ở đây là Ω = {T T, T G, GT, GG} với T là trai và G là gái. Và do vậy, dễ thấy xác suất T T là 1/4. Với TH2, Ω = {T T, T G} ứng với người đầu là con trai, và do vậy, xác suất T T xảy ra là 1/2. Với TH3, Ω = {T T, T G, GT}, nên T T xảy ra với xác suất 1/3, chính là bài toán ở đầu đề của chúng ta. Với TH4, lời giải khó hơn vì thông tin ngày thứ 3 dường như không liên quan. Trước tiên, giả sử mỗi người con đều thuộc và một trong hai nhóm nào đó, mà tần suất của nhóm P là p và tần suất của nhóm Q là q, với p + q = n với n là tổng khả năng có thể có. Ví dụ ở TH4, hai nhóm đó là P = "sinh vào ngày thứ ba" và Q = "không sinh vào ngày thứ ba" có p = 1 và q = 6. Như vậy, không gian mẫu Ω có 16 trường hợp ứng với tần suất xảy ra như sau: Hàng/Cột C1 C2 C3 C4 H1 H2 H3 H4 TP TP [p2] TP TQ [pq] TPGP [p2] TPGQ [pq] TQTP [pq] TQTQ [p2] TQGP [pq] TQGQ [q2] GP TP [p2] GP TQ [pq] GPGP [p2] GPGQ [pq] GQTP [pq] GQTQ [q2] GQGP [pq] GQGQ [q2] Qua bảng trên, ta thấy tổng tất cả các khả năng xảy ra là n4 và ứng với mỗi "khối" 2 × 2, số khả năng xảy ra là n2. Với bài toán của Martin (TH3), ta thấy đáp án chính là tổng số khả năng của 46 Tạp chí Epsilon, Số 09, 06/2016 khối 2 × 2 ở trên trái (là khối toàn T) chia cho toàn bộ khả năng của bảng, sau khi đã loại bỏ khối 2 × 2 ở góc dưới phải (tức là khối toàn G) nên xác suất xảy ra là n 3n2 =13. Quay lại bài toán ngày thứ ba, với ràng buộc khả năng P = "sinh vào ngày thứ ba" phải xảy ra, không gian mẫu Ω của bài toán sẽ là toàn bộ các ô ở hàng H1 và cột C1 của bảng, nghĩa là toàn bộ các ô có chứa Tp. Và yêu cầu của bài toán chính là các ô (H1, C1), (H1, C2) và (H2, C1). Xác suất (chúng tôi tạm ký hiệu là ρ), do vậy sẽ là: ρ =p2 + pq + pq p2 + pq + p2 + pq + pq + p2 + pq=p2 + 2pq 3p2 + 4pq=p + 2q 4n − p=2 −pn 3p + 4q=2n − p 4 −pn Khi p/n tiến dần tới 0, ρ sẽ bằng 1/2 và khi p/n tiến dần đến 1, ρ sẽ bằng 1/3. Với bài toán ngày thứ ba (TH4), p = 1 và n = 7, ta có xác suất bằng ρ =2−1/7 4−1/7 = 13/27. Và như vậy, yếu tố "ngày thứ ba", thật sự có ảnh hưởng đến kết quả cuối cùng! Chúng ta hãy thử cùng xét điều kiện sau: "có ít nhất một người là con trai và sinh vào ngày 13 tháng 6 và rơi vào thứ ba, xác suất để cả hai đều là con trai là bao nhiêu?" Bỏ qua năm nhuận, chúng ta có ρ =2−1/(7×365) 4−1/(7×365) ≈ 0.49995. 3. Cuối cùng thì đây có phải là nghịch lý? Như vậy, chúng ta đã cùng nhau giải quyết và có kết quả của câu hỏi: "Một gia đình có hai người con, biết rằng có ít nhất một người là con trai, hỏi khả năng cả 2 người đều là con trai là bao nhiêu" với đáp số là 1/3. Tuy nhiên, khi bài toán vừa ra đời, vẫn có nhiều tranh cãi, và chúng ta hãy thử cùng nhau khảo sát các phiên bản khác nhau của bài toán với cùng câu hỏi "xác suất để cả hai đều là con trai là bao nhiêu": "Trong tất cả các gia đình có hai con, mà trong đó ít nhất một người là con trai, một gia đình được chọn ngẫu nhiên để hỏi." Đáp án trong câu hỏi này là 1/3, cũng chính là câu hỏi gốc của Martin. "Trong tất cả các gia đình có hai con, một người con trong số đó được chọn ngẫu nhiên, và người đó là con trai." Đáp án lúc này lại là 1/2. Nói cách khác, ngoài vấn đề là một kết quả khác với trực quan thì cách phát biểu bài toán cũng rất dễ dẫn đến những bối rối gây tranh cãi. Quay lại bài toán ngày thứ ba, với yêu cầu "có ít nhất một người là con trai và sinh vào thứ ba" thì khả năng cả 2 đều là con trai là 13/27. Tuy nhiên, câu hỏi gốc của Foshee lại là: "Trong toàn bộ các gia đình có hai con mà ít nhất có một người là con trai và sinh vào ngày thứ ba, hãy cho biết tỉ lệ các gia đình mà cả hai người con đều là con trai." Và đáp án trong câu hỏi này, theo chính tác giả, là 1/2. 4. Lời kết Chuyên mục kỳ này ngắn hơn các kỳ trước do theo chủ ý của người viết, vốn dĩ các nghịch lý đã gây rất nhiều bối rối cho người đọc, nên nếu dùng quá nhiều bài toán sẽ khiến người đọc càng 47 Tạp chí Epsilon, Số 09, 06/2016 khó tiếp cận hơn, mất đi bản chất của toán học giải trí. Để có được bài viết này, chúng tôi tổng hợp và sử dụng các nguồn tham khảo chính sau: 1. Wikipedia, Boy or Girl Paradox. 2. Gardner, Martin, 1954: The Second Scientific American Book of Mathematical Puzzles and Diversions. Simon & Schuster. 3. Peter Lynch, The Two-Child Paradox: Dichotomy and Ambiguity, Irish Math. Soc. Bulletin 67 (2011), pp. 67-73. Chúng tôi kết thúc chuyên mục tại đây và hẹn gặp lại các bạn trong số tới. 48 PHÉP BIẾN ĐỔI FOURIER CÓ Ý NGHĨA VẬT LÝ GÌ? Job Bouwman (Trần Nam Dũng dịch) Hãy tưởng tượng bạn gõ một phím đàn dương cầm, khi đó dây đàn sẽ rung lên và tạo ra một sóng âm có dạng hình sin như ta vẫn biết trong lượng giác. Bây giờ tưởng tượng ba phím được nhấn cùng một lúc, làn sóng âm này sẽ không còn là hình sin nữa, mà là một cái gì đó hỗn độn và phức tạp hơn. Nhưng ẩn trong làn sóng âm thanh lộn xộn này chỉ là một mô hình đơn giản: Sau tất cả, hợp âm tạo ra này chỉ là ba phím đánh cùng lúc, và vì vậy sóng âm thanh lộn xộn chỉ là kết quả của sự tổng hợp của ba nốt (hay ba sóng sin). Và đó chính là một ví dụ của biến đổi Fourier. Trong số này, Epsilon tiếp tục giới thiệu với độc giả phần bình luận từ bài viết của tác giả của tác giả Job Bouwman đã được đăng ở Epsilon số 7: Phép biến đổi Fourier có ý nghĩa vật lý gì? 1. Stephen Scholnik Thực sự tôi sử dụng phép biến đổi Fourier ... rất nhiều. Đó là một công cụ toán học để phân tích tín hiệu, nó không có một sự tương tự chính xác về mặt vật lý (IMHO). Nhưng phải nói rằng: Sự tương tự tốt nhất mà tôi có thể nghĩ ra là nó cho ta biết giá trị tần số trung bình của một mẫu thử của, chẳng hạn như, một bản ghi âm. Như vậy nếu như đó là bản ghi âm của một bản nhạc, nói chung nó sẽ cho chúng ta biết năng lượng âm thanh trung bình (và pha) của mỗi nốt nhạc trong toàn bài. Nhưng điều đó cũng không đúng hẳn bởi vì mỗi nốt nhạc có sự hòa âm và phép biến đổi Fourier không thể phân biệt được sự hòa âm giữa nốt La và nốt La trong một quảng tám cao hơn. Ngoài ra, như tôi đã nói, phép biến đổi Fourier cho bạn biết pha trung bình của mỗi tần số mà điều này thông thường lại không có một minh họa vật lý nào cả (mặc dầu bạn có thể nghe được những biến đổi pha động). Tệ hơn nữa là, một nốt nhạc càng gõ mạnh (tấn công sắc bén hơn và rơi xuống), phổ tần số của nó càng mờ. Như vậy có những tần số trong phép biến đổi có năng lượng đáng kể nhưng lại không tương ứng với một nốt nhạc nào cả. Do tất cả những lý do đó, sự tương tự hầu như là một sai lầm (thêm vào sau đó). Tôi thích câu trả lời của Lionel Chiron. Ông ta đưa ra những ví dụ đích thực về những dữ liệu quan sát cụ thể tương ứng với phép biến đổi Fourier của một cái gì đó cụ thể. Ngoài ra câu trả lời của ông ta làm tôi nhớ rằng một bức ảnh toàn ký 3 chiều (thứ mà bất cứ ai đam mê khoa học đều biết và yêu thích) là một sự tương tự ở mức độ cao của phép biến đổi 2 chiều của cảnh mà nó xây dựng lại. 49 Tạp chí Epsilon, Số 09, 06/2016 2. Konstantinos Konstantinides, Kỹ sư điện làm việc trên lãnh vực xử lý tín hiệu và mã hóa video Một phép biến đổi Fourier (FT) một mình nó không có nghĩa gì cả. Đó là một hàm số. Đầu ra của phép biến đổi Fourier cho ta biễn của tín hiệu trên miền tần số. Có khi làm việc với miền thời gian tiện hơn, có khi làm việc với miền tần số tiện hơn. Chẳng hạn như, nếu có ai đó bảo với bạn rằng đây là một bộ “lọc thông thấp”, bạn có thể hiểu rằng bộ lọc này sẽ lọc bỏ tất cả những tần số cao sau một phép lọc tần, mặc dầu biểu diễn trong miền thời gian (đáp ứng xung) có thể khá phức tạp. Trong xử lý tín hiệu, tích chập trong miền thời gian tương ứng với phép nhân trong miền tần số. Điều này giúp ích rất nhiều khi bạn phân tích đáp ứng tần số của một hệ thống. Vậy là phép biến đổi Fourier có thể xem như một công cụ chuyển đổi qua lại giữa miền thời gian và miền tần số. 3. Samuel O. Ronsin, PhD, Toán ứng dụng Thêm một ý về “ý nghĩa vật lý”, thật thú vị là phép biến đổi Fourier (cụ thể là dưới dạng trực giao) là “căn bậc hai” của toán tử “đảo chiều” Φ : f 7→ Φ(f) = (x 7→ f(−x)) trong một không gian L2 nào đó, do vậy F ◦ F = Φ. Điều này có ý nghĩa gì? Vâng, nếu bạn muốn phân tích toán tử “đảo chiều” một hàm nào đó thành hai toán tử giống nhau, phép biến đổi Fourier chính là cái bạn muốn tìm. Dầu rằng còn có thêm một vài ví dụ nữa ... 4. Norman Corbett, PhD, Toán ứng dụng Đối với câu trả lời hiện có: Phép biến đổi Fourier được xây dựng nhờ những tần số điều hòa ... bội nguyên của một vài tần số cơ bản. Một trong những điểm tách rời với âm nhạc là ở chỗ các nốt nhạc tuân theo thang hàm mũ (hãy nghĩ về chiếc đàn piano của bạn). Điều này không có nghĩa là phổ của mọi quá trình vật lý đều bị ràng buộc theo kiểu như vậy. Bên cạnh đó, các tính chất toán học của phép biến đổi Fourier đảm bảo rằng nó có thể xử lý dữ liệu âm thanh, hình ảnh, vết đen trên mặt trời, ẩn tinh, ... và làm lộ ra những đặc điểm vật lý cơ bản. Phải chăng phép biến đổi Fourier có một nguyên nhân phát sinh cụ thể? Vâng! Joseph Fourier đã khởi đầu tất cả khi nghiên cứu về sự truyền nhiệt trong khí quyển. Ở đây phân tích tín hiệu ở tần số điều hòa là hết sức thõa đáng ... (vâng cho mô hình tuyến tính). Phép biến đổi Fourier có ý nghĩa vật lý gì không? Tất nhiên là có. Hãy xem xét nguyên nhân ban đầu thúc đẩy sự ra đời của nó. Nói rằng phép biến đổi Fourier chỉ đơn giản là một vấn đề trừu tượng cũng giống như nói Navier-Stokes không có cơ sở nào trong thực tại. 50 Tạp chí Epsilon, Số 09, 06/2016 5. Singer Karthik, Kỹ sư điện xử lý tín hiệu số và điện tử số Phép biến đổi Fourier (FT) là một mô hình/trừu tượng hóa toán học phân tích một tín hiệu thành những tín hiệu cơ bản cũng giống như một lăng kính nhận vào ánh sáng trắn và tách nó ra thành những tần số ánh sáng khác nhau. Không có sự tương tự nào tốt hơn. Trước hết, mỗi tín hiệu có một thành phần tần số chứ không phải là những đặc điểm của miền thời gian. Đa số những ai không phải là kỹ sư gặp khó khăn với khái niệm tần số. Hiểu khái niệm này trước và phép biến đổi Fourier sẽ trở nên dễ hiểu hơn. Nói tóm lại, phép biến đổi Fourier cho phép bạn phân tích tín hiệu trong một miền xác định khác giúp ta hiểu thấu những thành phần tần số cơ bản tạo thành tín hiệu mà ta nghiên cứu. Nếu như bạn thật sự muốn phân tích nội dung của một tín hiệu bạn phải xem xét nó trong miền tần số. Đồ thị biên độ và tần số cho ta biết tần số nào là chủ đạo. Từ đó bạn có thể xác định chẳng hạn một tần số xác định chứa nhiễu và thiết kế một bộ lọc để lọc nhiễu đó đi. Hoặc giả bạn muốn làm suy yếu những tần số xác định và khuếch đại những tần số còn lại. Nó giúp bạn tạo hình tín hiệu như mong muốn. 6. Jonathan Roberts Phép biến đổi Fourier là một phép biến đổi từ miền thời gian vào miền tần số. Nếu bạn áp dụng phép biến đổi Fourier cho một sóng cosine bạn sẽ có một đoạn thẳng đứng trên đồ thị. Mọi sóng phức tuần hoàn liên tục có thể tạo thành từ tổng của những chuỗi hình sin trực giao. Biến đổi của những sóng gián đoạn phát sinh lỗi gọi là hiện tượng Gibbs. Vài hình ảnh hữu ích Nguồn: Steve on Image Processing, Frequency domain 51 Tạp chí Epsilon, Số 09, 06/2016 7. Henk Mulder Phép biến đổi Fourier đo sự tương quan của một tín hiệu tuần hoàn và một chuỗi những hàm điều hòa hình sin. Đây là cách mà toán học làm theo đúng nghĩa đen: tính sự tương quan của mỗi hàm điều hòa đối với tín hiệu nhập vào. Điều này cụ thể có nghĩa là phép biến đổi Fourier đo xem tín hiệu tuần hoàn giống hàm điều hòa tới mức nào. Đối vói những hàm điều hòa chủ đạo, bạn có thể nhận ra. Nhưng nếu như có nhiều hàm điều hòa thì điều này sẽ khó khăn hơn. Đó chính là lý do tại sao bạn dùng phép biến đổi Fourier để có một bức tranh chính xác về sự hiện diện của tất cả các hàm điều hòa trong tín hiệu nhận vào. Cần nói thêm là thật thú vị khi người chị em của phép biến đổi Fourier, phép biến đổi Laplace thực hiện chính xác cách làm đó với những hàm mũ. Nó tính tương quan của hàm mũ đối với những thành phần của hàm đầu vào và biểu diễn hàm này dưới dạng những hàm mũ. Vì hàm mũ có đạo hàm là chính nó, điều này cho phép bạn xử lý dễ dàng nhiều phương trình vi phân. 8. Michael Kownacki Làm việc trong một xí nghiệp làm giàu uranium, tôi thường xuyên dùng phép biến đổi Fourier để lấy giá trị định lượng trong việc xử lý sự cố những máy móc đồ sộ sử dụng đầu ra của gia tốc kế. Mỗi vấn đề hiện ra như một tần số đặc biệt, shaft alignment, sự rung của chong chóng turbine, ... Bằng cách lấy tích phân bình phương của phép biến đổi Fourier trong một vùng tần số đặc biệt, tôi có thể tính được năng lượng của mỗi vấn đề cụ thể. Tôi cũng có thể xác định được chuyển động rối làm giảm hiệu quả của cả quá trình. Tất cả những thứ bạn cần làm là lắc nhẹ một cái van đầu vào để thay đổi phân bố áp suất làm cho chuyển động rối biến mất. Một khi sự rung trong vùng rối biến mất, bạn biết rằng van được bố trí đúng. Một vấn đề nghiêm trọng xảy ra với turbine là khi áp suất không đều chung quanh chu vi của nó (thường gọi là đột biến thứ cấp). Nó làm cho chong chóng turbine chết máy mỗi khi chúng quay, tạo thành sự rung chong chóng có thể làm gãy chong chóng. Đối với những máy lớn có tốc độ cao, điều này có thể khiến cho toàn bộ cái máy nổ tung đúng theo nghĩa đen. 9. Mil Ford Tôi nghĩ rằng một ví dụ trong đề tài của tôi, một phần trong đó là xác định độ cao của thủy triều có thể bổ ích cho các bạn. Thủy triều xuất hiện do lực hấp dẫn của mặt trời, mặt trăng, ... Vì mặt trăng quay chung quanh quả đất và quả đất quay chung quanh mặt trời, chiều cao của thủy triều có dạng hàm sin / cos Tôi dùng một phương pháp gọi là Phân Tích Fourier Rời Rạc tạo thành trước Phân Tích Fourier dựa trên những điểm dữ liệu thay vì hàm. Về căn bản chúng ta sẽ so trùng một lô những đường cong sin / cos, như là phép hồi quy phi tuyến tính đối với các điểm dữ liệu và biến đổi nó thành tần số ở đầu ra (đồ thị vẽ cái bạn nhận được sau cùng). 52 Tạp chí Epsilon, Số 09, 06/2016 Kết quả là, chẳng hạn một tần số nửa ngày, chúng ta biết rằng hiệu ứng thủy triều là do sự quay của trái đất, biết tần số chúng ta dễ dàng tìm ra biên độ Cn. Từ đó bạn có thể viết thành một hàm dài chẳng hạn như C1 · sin(· · ·) + C2 · sin(· · ·) Vẽ hàm này ra giúp ta dự đoán thủy triều trong tương lai. Tiến hành phép biến đổi bạn căn bản làm khi chuyển từ cõi thời gian sang không gian tần số. 10. David Tung, một nhà doanh nghiệp Phép biến đổi Fourier kết hợp hai khái niệm vật lý: Tuần hoàn và cơ sở (đó là một tín hiệu tốt có thể phân tích thành các thành phần độc lập và các thành phần này có thể kết hợp lại tạo thành nguyên tín hiệu ban đầu). Nhưng phép biến đổi Fourier tạo ra khái niệm tần số, một phần trong sự hiểu biết của chúng ta về thế giới vật lý, như vậy có thể cũng đúng khi nói rằng “tìm tần số tương ứng với tin hiệu chính là ý nghĩa vật lý của phép biến đổi Fourier”, nếu chúng ta quên đi vấn đề con-gà-và-quả-trứng. 11. Rod Schmidt Trong tai trong có ốc tai cuộn xoắn lại. Tưởng tượng nó không xoắn nữa và trải dài ra. Nó là một hình nón dài và hẹp. Khi âm thanh vào tai, những tần số thấp hơn (bước sóng dài nhất) tạo thành cộng hưởng rung gần đầu lớn mở của hình nón. Những tần số cao (bước sóng ngắn nhất) gây ra những cộng hưởng rung tương tự gần đỉnh của hình nón. Như vậy các tần số được phân loại. Mỗi tần số tương ứng với một khoảng cách dọc theo hình nón. Đối với phép biến đổi Fourier ngược ... hãy hình dung ra đúng điều đó, chỉ có điều bộ phim được chiếu ngược lại. 12. Lionel Chiron Liên quan với câu hỏi của bạn, phép biến đổi Fourier có thể xem như một hiện tượng tự nhiên.. chẳng hạn nó là trường hợp của tinh thể học khi những đốm quan sát được là phép biến đổi Fourier của cấu trúc tinh thể dưới phép rọi X-quang, đó là trường hợp của MRI khi tín hiệu đăng ký (k-không gian) là phép biến đổi Fourier trực tiếp của hình ảnh mà chúng ta muốn khám phá, đó là trường hợp của Quang Học Fourier: hiện tượng nhiễu xạ sinh ra (trong một chừng mực gần đúng phụ thuộc vào khoảng cách) một phép biến đổi Fourier của hình ảnh trước màn chắn. Điều này là khả dĩ vì tất cả những hiện tượng nói trên dựa trên một sự kiện là có những hiện tượng thiên nhiên có thể được mô hình hóa một cách chính xác nhờ một tần số và một biên độ ... Tổng quát hơn Cơ Học Lượng Tử sử dụng khái niệm phép quay trong không gian trừu tượng và can thiệp nhiều hơn so với vật lý cổ điển ... và do vậy sự xuất hiện tự nhiên của phép biến đổi Fourier vốn mang lại một cách nhìn mới của cùng một hiện tượng lại càng “tự nhiên” hơn nữa ... Với Cơ Học Lượng Tử, khái niệm tần số quả thật đã mở rộng sang cơ học ... Rõ ràng nó không chỉ giới hạn trong quang học và âm thanh ... 53 Tạp chí Epsilon, Số 09, 06/2016 54 TẬP HỢP TRÙ MẬT VÀ ỨNG DỤNG Kiều Đình Minh, Nguyễn Quang Khải (THPT chuyên Hùng Vương, Phú Thọ) Trong bài viết này, các tập hợp được hiểu là tập con của R. 1. Tập hợp trù mật Định nghĩa 1. Cho hai tập hợp A, X với A ⊂ X. Ta nói rằng A trù mật trong X nếu với mọi x ∈ X và mọi ε > 0 tồn tại x ∈ A sao cho |x − x| < ε. Một cách tương đương, với mọi x, y ∈ X, x < y tồn tại x ∈ A sao cho x < x < y. Nói một cách nôm na rằng, tập A trù mật trong X nếu mọi phần tử trong X đều bị kiểm soát bởi những phần tử trong A với khoảng cách bé tuỳ ý. Định lý 1.1. Tập hợp A trù mật trong tập hợp X khi và chỉ khi với mọi x ∈ X, tồn tại dãy (xn) thoả mãn xn ∈ A, ∀n ∈ N và lim n→+∞xn = xA = X . Chứng minh. Thật vậy, nếu x ∈ A thì chọn xn = x, ∀n. Nếu x /∈ A, theo định nghĩa ta có: Với mọi x ∈ X và mọi ε > 0, tồn tại x ∈ A sao cho |x − x| < ε. Mặt khác dễ thấy tồn tại dãy (εn) thoả mãn 0 < εn < ε, ∀n ∈ N sao cho mỗi n ∈ N, tồn tại xn ∈ A thoả mãn |xn − x| < εn. Suy ra |xn − x| = |xn − x + x − x| 6 |xn − x| + |x − x| < εn + ε < 2ε, do đó lim n→+∞xn = x. Ngược lại là hiển nhiên. Nhận xét. Nếu A trù mật trong X thì với mọi x ∈ X, tồn tại hai dãy (xn1),(xn2) ∈ A thoả mãn: (i) xn1 < x < xn2, ∀n1, n2 ∈ N. (ii) lim n1→+∞xn1 = lim n2→+∞xn2 = x. Định lý 1.2. Tập số hữu tỷ Q là trù mật trong tập số thực R. Chứng minh 1. Ta có nguyên lý Archimet “với x, y ∈ R sao cho x > 0, y > 0 thì luôn tồn tại n ∈ N sao cho y 6 nx”. Xét x, y ∈ R, x < y. Đặt d = y − x > 0. Theo nguyên lý Archimet tồn tại n ∈ N∗sao cho nd > 1, suy ra 1n < d. Gọi m = [nx] + 1 thì m − 1 6 nx < m suy ra nx < m 6 nx + 1, suy ra tiếp x < mn 6 x +1n < x + d = y. Chứng minh 2. Ta cần chỉ ra ∀a, b ∈ R, ∃mn(m ∈ Z, n ∈ Z+) sao cho a < mn < b. Thật vậy, chọn n ∈ Z+ sao cho đoạn [na, nb] có độ dài lớn hơn 1 hay nb − na > 1 tức n > 1 tồn tại m ∈ (na, nb), suy ra na < m < nb suy ra a < mn < b. 55 b−a. Khi đó Tạp chí Epsilon, Số 09, 06/2016 Chứng minh 3. Không mất tổng quát, ta cố định b và đặt b−a = ε. Xét tập A = {x ∈ Q\x < b}. Theo định nghĩa về Supremum: ∀ε > 0, ∃x0 sao cho b − ε < x0 < b. Như vậy ta chỉ cần chứng minh SupA = b. Gọi SupA = s. Xét n ∈ N∗ta chứng minh s −1n6 b 6 s +1n. Bất đẳng thức bên trái là hiển nhiên vì ∃x∗sao cho s −1n < x∗ < b. Giả sử s +1n < b. Nếu s ∈ Q thì s +1n∈ A và s +1n > s (vô lý). Nếu s /∈ Q thì w =[(n + 1) s] n + 1+1 n + 1∈ Q, và thoả mãn s < w < s +1n. Do đó w < b hay w ∈ A (vô lý). Suy ra |b − s| 61nsuy ra tiếp lim |b − s| = 0 tức b = s. Chứng minh 4. Kí hiệu [a] là phần nguyên của a. Ta có [a] 6 a < [a] + 1 ⇒[a]a6 1 <[a] + 1 a(a > 0). Theo định lý giới hạn kẹp ta được lim a→+∞[a]a = 1 = lim a→−∞[a]+1 a. Cho x là số thực dương, với mọi n ∈ N∗, đặt xn =[10nx] 10n và x0n =[10nx]+1 10n . Hai dãy (xn) và (x0n) là các giá trị hữu tỉ gần đúng dưới và gần đúng trên của x với độ chính xác 10−n. Dễ thấy xn, x0n ∈ Q và (xn) tăng còn (x0n) giảm. Ngoài ra lim n→+∞xn = x và lim n→+∞x0n = x. Nếu x < 0 thì hai dãy số lập như trên sẽ tiến tới −x. Bởi vậy hai dãy số (−xn) và (−x0n) sẽ tiến tới x. Chứng minh 5. Xét một số thực x nào đó. Ta chứng minh rằng ∀ε > 0, ∃r ∈ Q : |r − x| < ε. Thật vậy, nếu x ∈ Q thì ta chỉ cần chọn r = x. Ngược lại, x /∈ Q, xét biểu diễn thập phân vô hạn không tuần hoàn của x, x = a0a1a2 · · · an · · · a∞. Ta thấy ∀ε > 0, ∃n ∈ N : ε > 10−n. Khi đó, chọn r = a0a1a2 · · · an+1 ∈ Q ta thấy ngay |r − x| < 10−n < ε. Chú ý. Tập số vô tỉ là trù mật trong tập số thực R. Thật vậy, với mọi a, b ∈ Q, a < b. Đặt e = b − a > 0, luôn tìm được số tự nhiên n đủ lớn sao cho ne > 2. Khi đó n (b − a) > 2, suy ra nb > 2 + na. Chọn q =√2 + na là số vô tỉ, do √2 là số vô tỉ và na là số hữu tỉ. Ta có na < √2 + na < 2 + na < nb ⇒ na < q < nb ⇒ a n2) và m1, m2sao cho các số n1θ + m1 và n2θ + m2 thuộc cùng một khoảng ∆k nào đó. Ta có 0 < (n1 − n2) θ + (m1 − m2) < ε. 56 Tạp chí Epsilon, Số 09, 06/2016 Bây giờ ta chọn ε thoả mãn ε < b−a 2. Ta có b − a (n1 − n2) θ + (m1 − m2)>b − a ε> 2. Do đó sẽ tồn tại một số nguyên k nằm giữa a (n1−n2)θ+(m1−m2)và b (n1−n2)θ+(m1−m2), tức là a < k (n1 − n2) θ + (m1 − m2) < b. Bây giờ chọn N = k (n1 − n2) thì số {Nθ} ∈ (a; b). Vậy ta kết luận {{nθ} | n ∈ Z+} trù mật trong (0; 1). Định lý được chứng minh xong. Định lý 1.4. (Thác triển hàm liên tục) Cho hai hàm số f, g : X → X, trong đó f liên tục, g liên tục hoặc đơn điệu và A là tập trù mật trong X. Khi đó nếu f (x) = g (x), ∀x ∈ A thì f (x) = g (x), ∀x ∈ X. Chứng minh. Ta xét hai trường hợp: a) g là hàm liên tục: Theo định lý 1 thì ∀x ∈ X, ∃(xn)n>0 ∈ A : lim n→+∞xn = x. Ta có f (xn) = g (xn), ∀n ∈ N suy ra lim n→+∞f (xn) = lim n→+∞g (xn) ⇒ f (x) = g (x). b) g là hàm đơn điệu: Không mất tổng quát, giả sử g là đơn điệu tăng. ∀x ∈ X, ∃xn1, xn2 ∈ A sao cho xn1 < x < xn2 và lim n2→+∞xn1 = lim n2→+∞xn2 = x. Ta có f (xn1) = g (xn1) < g (x) < g (xn2) = f (xn2). Chuyển qua giới hạn ta được f (x) = g (x). Trong mọi trường hợp định lý đều được chứng minh. Nhận xét. Từ định lý 2 và định lý 4 thì ta có nghiệm của phương trình hàm trên R khi biết hàm đó trên Q và tính liên tục hoặc đơn điệu trên R. Tiêu biểu là lớp phương trình hàm Cauchy. 2. Một số ví dụ minh họa Ví dụ 1. Cho dãy số thực dương (an), tăng và không bị chặn. Chứng minh rằng tập hợp n S1 = m an| m, n ∈ N∗ o trù mật trong R+. Chứng minh. Nhận xét rằng với mọi số α > 0 và dãy (an) có tính chất lim n→+∞an = +∞ thì cũng có lim n→+∞αan = +∞. Xét p, q ∈ R+ thoả mãn p > q. Khi ấy lim n→+∞(p − q) an = +∞. Do đó, tồn tại n0 ∈ N∗sao cho (p − q) an0 > 2 ⇒ pan0 > [pan0 − 1] > qan0. Chọn m0 = [pan0] − 1, ta có p > m0 an0> q. Vậy S1 trù mật trong R+. m|m, n ∈ N∗ trù mật trong R+. Nhận xét. Từ ví dụ này suy ra các tập m2n | m, n ∈ N∗ , an 57 Tạp chí Epsilon, Số 09, 06/2016 Ví dụ 2. Giả sử N∗ được chia thành hai tập con A và B, mỗi tập chứa vô hạn phần tử. Đặt S3 = ab| a ∈ A, b ∈ B . Chứng minh rằng tập S3 trù mật trong [0; +∞). Chứng minh. Do B có vô hạn phần tử nên ∀k ∈ N, ∀ε > 0, ∃b ∈ B sao cho b > k2ε. Do A có vô hạn phần tử nên b (x + ε) > b (x − ε) + k, ∀x ∈ [0; +∞) ⇒ ∃a ∈ A : b (x + ε) > a > b (x − ε), suy ra x + ε >ab> x − ε ⇒ x −ab < ε. Vậy S3 trù mật trong [0; +∞). Nhận xét. Nếu A trù mật trong R+ thì B = 1r| r ∈ A cũng trù mật trong R+. Ví dụ 3. Cho (an)n>1là dãy số nguyên dương thoả mãn 0 < an+1 − an <√an, ∀n ∈ N∗. Đặt S4 = n ak al| k, l ∈ N∗, k > l o . Chứng minh rằng tập S4 là trù mật trong (0; 1). Chứng minh. Ta chứng minh nhận định sau: ∀ 0 < x < y < 1, ∃k, l ∈ N∗, k > l : x 0, ∃l ∈ N∗sao cho a1 √al< y − x. Xét các phần tử của dãy số hữu hạn a1 al 0 sao cho p−q Chứng minh. Xét p, q ∈ R+, p > q. Do lim n→+∞an có, tồn tại N sao cho p+q· k > ε. Ta an n− k Đặt α =q

N ⇒2q p + q· k + ε N. p+qk+ε

N và m0 sao cho α 0, tồn tại phần tử Rn sao cho 0 < Rn − α <1q2n. (2.3) Thực vậy, lấy một số n lẻ và chú ý rằng (qnxn+1 + qn−1) qn > q2n. Vì lim n→+∞qn = +∞, với n đủ lớn ta có 1qn< ε. Từ điều này và (2.3) suy ra 0 < pn − αqn <1qn< ε với n đủ lớn, do đó tồn tại n0 ∈ N sao cho n0 (pn − αqn) ∈ (a; b). Cho t ∈ [−1; 1], tồn tại số thực x sao cho t = sin x. Từ nhận xét ở trên suy ra tồn tại các dãy các số nguyên dương {mn} và {kn} với x = lim n→+∞(mn − 2πkn), Sử dụng tính liên tục của hàm sin x ta được t = sin x = sin lim n→+∞(mn − 2πkn) = lim n→+∞sin mn. Vậy mọi số thuộc khoảng [−1; 1] đều là điểm giới hạn của tập S6 = {sin n | n ∈ N}. Ví dụ 6. Cho u, v ∈ R+. Chứng minh rằng tập hợp {au + bv | a, b ∈ Z} trù mật trong R khi và chỉ khi uvlà một số vô tỷ. Chứng minh. Ta có {au + bv | a, b ∈ Z} trù mật trong R nên vauv + b | a, b ∈ Z trù mật trong R do đó auv + b | a, b ∈ Z trù mật trong R suy ra auv | a ∈ Z trù mật trong (0; 1) vì thế uvlà số vô tỷ. Nhận xét. Với u 6= 0, v 6= 0 thì bài toán vẫn đúng. Ví dụ 7. Cho các số thực dương a, b thoả mãn b [an] = a [bn] , ∀n ∈ Z+. Chứng minh rằng a = b hoặc cả a và b đều nguyên. 59 Tạp chí Epsilon, Số 09, 06/2016 Chứng minh. Giả sử a 6= b và cả a, b đều vô tỷ, ta có b [an] = a [bn] ⇔ ban − b {an} = abn − a {bn} ⇔ b {an} = a {bn} . (2.4) Không mất tổng quát, giả sử a > b, ta có a={bn} b {an}, ∀n ∈ Z+ ⇒ba> {bn} , ∀n ∈ Z+, điều này vô lý do b là số vô tỷ nên {{bn} | n ∈ Z+} là trù mật trong (0; 1). Do đó một trong hai số a, b là hữu tỷ. Giả sử a ∈ Q, khi đó tồn tại n0 ∈ Z+ sao cho an0 ∈ Z+ ⇒ {an0} = 0. Do đó {bn0} = 0 ⇒ b ∈ Q. Đặt a =pq, (p, q) = 1 và b =st, (s, t) = 1. Trong (2.4) cho n = q có 0 = a {bq} ⇒ {bq} = 0 ⇒ {bq} ∈ Z+ ⇒ q...t. Tương tự, ta có t... q. Vậy q = t, do đó (2.4) tương đương với s p qn = p s qn , ∀n ∈ Z+. Với q > 1, ta chọn n ≡1s(mod q), suy ra sn q = 1 q =1q⇒ s pqn = p 1q =pq. Đặt pn = qk + r ⇒ s pn q = s ·rq⇒pq= s.rq⇒ p = s · r ⇒ p... s. Tương tự có s... p. Do đó p = s, vô lý do a 6= b, do vậy q = 1 và a, b ∈ Z. Tóm lại hoặc a = b hoặc a, b ∈ Z. Ví dụ 8. Cho a, b là các số thực dương thoả mãn [2an + b] chia hết cho 2 với mọi số nguyên dương n. Chứng minh rằng a là số nguyên dương. Chứng minh. Nếu a là số vô tỷ thì ta có [2an + b] = [2 {an} + b] + 2 [an]... 2, ∀n ∈ Z+ ⇒ [2 {an} + b]... 2, ∀n ∈ Z+. (2.5) Lại do {{an} | n ∈ Z+} là trù mật trong (0; 1). Suy ra tồn tại số nguyên dương n mà [b + 2 {an}] không chia hết cho 2. Thật vậy: Nếu 2k + 1 > b > 2k thì chọn n sao cho 2k + 2 > 2 {an} + b > 2k + 1. Nếu 2k + 2 > b > 2k + 1 thì chọn n sao cho 2k + 2 > 2 {an} + b > 2k + 1. Điều này mâu thuẫn với (2.5) nên suy ra a phải là số hữu tỷ. Nếu a không nguyên thì đặt a =pq, (p, q) = 1, q ∈ Z+, q > 1. Khi đó viết n = qt + r, 0 6 r 6 q − 1 thì [2an + b] = 2 pt +prq + b ... 2, ∀n ∈ Z+. 60 Tạp chí Epsilon, Số 09, 06/2016 Chọn r = 0 thì [2pt + b]... 2 ⇒ [b]... 2. Do đó [2an + b] = 2pt + [b] + 2pr q+ {b} ... 2, ∀t ∈ Z+, tương đương với 2pr q+ {b} ... 2, ∀t ∈ Z+, hay là 2 pr q + {b} ... 2, ∀t ∈ Z+. n Mà 3 > 2 pr q o + {b} > 0 suy ra h 2 n pr q o + {b} i ∈ {0; 2} . Ta xét hai trường hợp sau: • Nếu tồn tại r0 6= 0 sao cho h 2 n pr0 q o + {b} i = 0, thì 1 > 2 pr0 q + {b} > 0 ⇒12> pr0 q ⇒ p2r0 q = 2pr0 q = 2 pr0 q , và 2 p2r0 q + {b} = 4 pr0 q + {b} > 2 pr0 q + {b} . Cứ làm như vậy suy ra tồn tại số nguyên dương i mà (vì mỗi lần thực hiện thì giá trị biểu thức tăng lên) 2 p · 2i· r0 q + {b} < 1  2 + {b} > 1 Khi đó p · 2i+1· r0 q 2 p · 2i+1· r0 q + {b} = 4 p · 2i· r0 q + {b} < 2 2 p · 2i· r0 q + {b} < 2. Do đó 2 p · 2i+1· r0 q + {b} = 1. Gọi r1 là số dư 2i+1· r0 khi chia cho q, suy ra vô lý. h • Nếu không tồn tại r0 6= 0 sao cho 2 n pr0 q o + {b} i = 0, thì với mọi r0 6= 0 ta có 2 + {b} > 2, mà pr0 q + {b} = 2 ⇒ 3 > 2 pr0 q 1 > {b} > 0 ⇒ 2 pr0 q > 1 ⇒ pr0 q 61 >12⇒ p · 2r0 q = 2 ·pr0 q = 2 pr0 q −1 < pr0 q . Tạp chí Epsilon, Số 09, 06/2016 Cứ làm như vậy, suy ra tồn tại số nguyên dương i sao cho (vì mỗi lần thực hiện thì giá trị biểu thức giảm xuống) 2 pr0 · 2i q + {b} > 2 + {b} < 2 n Do 2 pr0·2i q  o + {b} > 2, nên 2 pr0 · 2i+1 q 2 pr0 · 2i+1 q +{b} = 2 2 pr0 · 2i q − 1 +{b} = 4 pr0 · 2i q +{b}−2 > 2 pr0 · 2i q > 1. Suy ra h 2 n pr0·2i+1 q o + {b} i = 1. Chọn r1 là số dư khi chia r02i+1 cho q suy ra điều vô lý. Tóm lại a ∈ Z+. Ví dụ 9. Đặt A = 2m 3n | m, n ∈ Z+, 2m > 3n . Chứng minh rằng inf(A) = 1. Chứng minh. Ta đặt B = {log3a | a ∈ A} . Khi đó ta phải chứng minh inf (B) = log31 = 0. Ta thấy b > 0, ∀b ∈ B suy ra inf(B) > 0. Lại thấy với mỗi b ∈ B, b có dạng nlog32 − m. Với mỗi số nguyên dương n mà n · log32 > 1, xét m = [nlog32] ∈ Z+. Khi đó, b = {nlog32}. Lại thấy log32 là số vô tỷ dương nên theo định lý Kronecker thì {{nlog32} |n ∈ Z+} trù mật trong (0; 1). Do đó tồn tại dãy số nguyên dương ni để bi = {nilog32} → 0, khi i → +∞. Do vậy inf (B) = 0, suy ra inf(A) = 1. Ví dụ 10. (Putnam 1995) Cho A (x) = {[nx] |n ∈ Z+} . Chứng minh rằng nếu a, b, c ∈ R+ thì A (a), A (b), A (c), không thể là một phân hoạch của Z+. Chứng minh. Giả sử phản chứng nếu a, b, c ∈ R+ thì A (a), A (b), A (c) là một phân hoạch của Z+. Ta xét các trường hợp sau: • Trong các số a, b, c có ít nhất một số hữu tỷ, giả sử là a. Đặt a =pq, (p, q) = 1, q ∈ Z+. Ta chứng minh tồn tại các số nguyên dương m, n sao cho [ma] = [nb] . (2.6) Nếu b ∈ Q thì tồn tại m, n ∈ Z+ mà (ma, nb ∈ Z+ ma = nb Khi đó ta có điều phải chứng minh. Nếu b 6∈ Q thì chọn m = qt, t ∈ Z+. Khi đó [ma] = h qt pq i = pt. Do b /∈ Q nên theo định lý Kronecker, tồn tại n0 ∈ Z+ sao cho {n0b} <1p. Khi đó [(pn0) b] = [p [n0b] + p {n0b}] = p [n0b] + [p {n0b}] = p [n0b] . Chọn t = [n0b] thì ta có điều phải chứng minh. Vậy (2.6) đúng và do đó A (a)∩ A (b) 6= ∅, trái với giả thiết phản chứng. 62 Tạp chí Epsilon, Số 09, 06/2016 • Cả ba số a, b, c đều là vô tỷ. Xét n ∈ Z+ bất kỳ, khi đó số các số thuộc A (a) mà nhỏ hơn hay bằng n là số các số m ∈ Z+ thoả mãn [am] 6 n ⇔ am − 1 < n ⇔ m 1 nên a a−1∈ I+ và a a−1 >32. Khi đó A (a), A a . một phân hoạch của Z+. Khi đó A (b) ∪ A (c) = Aa a−1 a−1 Nhận xét. Cho α ∈ I, α > 32và β ∈ R+ sao cho A (β) ⊆ A (α), khi đó βα∈ Z+. Chứng minh. Do A (β) ⊆ A (α) nên với mỗi m ∈ Z+ thì tồn tại n ∈ Z+ thoả mãn [mβ] = [nα] ⇒ nα − 1 < [mβ] < nα ⇒[mβ] α< n <[mβ] α+1α. (2.7) Ta thấy nếu tồn tại m ∈ Z+ mà 0 1−1α, ∀m ∈ Z+ ⇔ mβ − {mβ} α > 1−1α⇔ mx −{mβ} α > 1−1α, x =βα⇔ {mx} − {mβ} α Nếu x ∈ I, khi đó với mỗi m ∈ Z+ có > 1 −1α, ∀m ∈ Z+. (2.8)  α> 1 −1α   {mx} > 1 −1α+{mβ} {mx} − {mβ} α> 1 −1α⇒ − {mx} +{mβ} α> 1 −1α α+1α− 1 > {mx}⇒ {mβ} {mx} > 1 −1α 2 α− 1 > {mx} 1) Nếu 2α − 1 6 0 thì ta phải có {mx} > 1 −1α, ∀m ∈ Z+, trái với định lý Kronecker. 2) Nếu 2α − 1 > 0 thì ta có 1 −1α >2α − 1 do α > 32, suy ra với mọi m ∈ Z+ thì {mx} ∈/2α − 1; 1 −1α trái với định lý Kronecker. 63 Tạp chí Epsilon, Số 09, 06/2016 Do đó x /∈ I suy ra x ∈ Q. Nếu x /∈ Z+, đặt x =pq, (p, q) = 1, q > 1. Xét m = qk + 1, k ∈ Z+, khi đó {mx} − {mβ} α = (qk + 1) pq −{(qk + 1) β} α = p q −{k · qβ + β} α (2.9) = {x} − {kp · α + β} α . Vì α ∈ I nên pα ∈ I, do đó {{k (pα)} | k ∈ Z+} trù mật trên (0; 1), từ đó suy ra {{k (pα) + β} | k ∈ Z+} , trù mật trên (0; 1), dẫn đến n{k(pα)+β} α| k ∈ Z+ o trù mật trên 0; 1α . Suy ra tồn tại k0 ∈ Z+ sao cho α> {x}+1α−1 do 1 > {x} > {x} +1α− 1;1α> {x} +1α− 1 . 1 > {x} >{k0pα + β} Khi đó, từ (2.7) xét m = qk0 + 1, ta có {mx} − {mβ} α = {x} − {k0pα + β} α = {x} − {k0pα + β} < {x} − α {x} +1α− 1 = 1 −1α, trái với (2.9) Do đó x phải thuộc Z+, hay βα∈ Z+ và nhận xét được chứng minh. Trở lại với bài toán, do , mà a A (b) ∪ A (c) = A a a − 1 ⇒ A (b), A (c) ⊂ A a a − 1 a−1 >32nên tồn tại b0 ∈ Z+ sao cho b =a a−1· b0 và c0 ∈ Z+ sao cho c =a d =a a−1· b0c0 thìA (d) ⊂ A (b), A (d) ⊂ A (c), điều này vô lý do A (b) ∩ A (c) = ∅. Vậy điều giả sử ban đầu là sai, do đó ta có điều phải chứng minh. a−1· c0. Đặt Nhận xét. Cho ai ∈ R+ (i = 1, 2, . . . , n) đôi một phân biệt, khi đó nếu A (a1), A (a2), . . . , A (an) là một phân hoạch của Z+ thì n ∈ {1; 2} . Tiếp theo chúng ta cùng nghiên cứu một số bài toán mà cách giải được dùng phương pháp của tập trù mật. Ví dụ 11. (France TST 2005) Cho A ⊂ N∗, A 6= ∅ sao cho ∀x ∈ A thì 4x, [√x] ∈ A. Chứng minh rằng A = N∗. 64 Tạp chí Epsilon, Số 09, 06/2016 Chứng minh. Ta gọi a0 là số nguyên dương bé nhất thuộc A. Nếu a0 > 1 thì √a0 ∈ A và √a0 6√a0 < a0, mâu thuẫn với cách chọn a0 vì vậy a0 = 1. Từ giả thiết suy ra 4n ∈ A, ∀n ⇒h√4ni= 2n ∈ A, ∀n ⇒h(2n)1 2m i ∈ A, ∀m, n. (2.10) Ta có nhận xét sau ∀k ∈ N∗, ∃m, n ∈ N∗: k2m6 2n < (k + 1)2m. Thật vậy, chọn m ∈ N∗ để 2m [log2(k + 1) − log2k] > 2, khi đó log2(k + 1)2m− log2k2m> 2 ⇒ ∃n ∈ N∗: log2k2m6 n < log2(k + 1)2m, suy ra k2m6 2n < (k + 1)2m ⇒ k =h(2n)1 2m i . Từ (2.10) suy ra k ∈ A nên N∗ ⊂ A. Do đó A = N∗. Ví dụ 12. (VMO 1997, bảng A) Cho số tự nhiên n > 1 không chia hết cho 1997. Xét hai dãy số (ai) và (bj ) được xác định như sau: ai = i +ni 1997, i = 1, 2, . . . , 1996, bj = j +1997j n, j = 1, 2, . . . , n − 1. Xét tất cả các số của hai dãy trên và sắp thứ tự không giảm ta được c1 6 c2 6 · · · 6 c1995+n. Chứng minh rằng ck+1 − ck < 2, ∀k = 1, 2, . . . , 1994 + n. Chứng minh. Thay 1997 bởi số m không là ước của n. Xét hai dãy số 1 +nm , i = 0, 1, . . . , m, bj = j 1 +mn , j = 0, 1, . . . , n. Ta có ai = i a0 = 0 < a1 < · · · < am = m + n, b0 = 0 < b1 < · · · < bn = m + n. Nếu n < m thì ai+1 − ai = 1 + nm < 2. Với mỗi k mà 1 6 k 6 m + n − 2 thì tồn tại duy nhất j để aj 6 ck < aj+1 với 0 6 j 6 n − 1. Khi đó ck+1 6 aj+1 và ck+1 − ck 6 aj+1 − aj < 2. Nếu m < n thì bj+1 − bj = 1 + mn < 2. Với mỗi k mà 1 6 k 6 m + n − 2 thì tồn tại duy nhất j để bj 6 ck < bj+1 với 0 6 j 6 n − 1. Khi đó ck+1 6 bj+1 và ck+1 − ck 6 bj+1 − bj < 2. Ta có điều phải chứng minh. Ví dụ 13. Với α > 1 là một số thực sao cho ∀m, n ∈ N∗, m... n ⇒ [mα]... [nα]. Chứng minh α ∈ N∗. Chứng minh. Giả sử α = t + r, t ∈ N∗, 0 6 r 6 1. Từ giả thiết ∀k, n ∈ N∗sao cho [knα]... [nα] , ta có [knα] = [knt + knr] = k [nt] + [knr] ≡ [knr] (mod nt), suy ra ∀k, n ∈ N∗. Lại có [knr]... [nα] . Cố định k, cho n → +∞ có nt → +∞ mà [knr] bị chặn nên suy ra {nr} = 0 ⇒ r = 0 ⇒ α = t ∈ N∗. Ta có điều phải chứng minh. 65 Tạp chí Epsilon, Số 09, 06/2016 Ví dụ 14. (IMO 1997) Cho x1, x2, . . . , xn ∈ R thoả mãn Xn i=1 xi = 1, |xi| 6n + 1 2, ∀1 6 i 6 n. Chứng minh rằng tồn tại một hoán vị (y1, y2, . . . , yn) của (x1, x2, . . . , xn) sao cho Xn i=1 iyi 6n + 1 2. Chứng minh. Với mọi hoán vị Π = (y1, y2, . . . , yn) của (x1, x2, ..., xn), ta kí hiệu S (Π) là giá trị của tổng y1 + 2y2 + · · · + nyn. Đặt r =n+1 2. Ta cần phải chứng minh rằng tồn tại Π nào đó để |S (Π)| 6 r. Gọi Π0 là hoán vị đồng nhất Π0 = (x1, x2, . . . , xn) và gọi Π là hoán vị đảo ngược Π = (xn, xn−1, . . . , x1). Nếu |S (Π0)| 6 r hoặc SΠ 6 r, bài toán coi như được giải xong, do vậy, ta giả sử |S (Π0)| > r và SΠ > r. Để ý S (Π0) + SΠ = (x1 + 2x2 + · · · + nxn) + (xn + 2xn−1 + · · · + nx1) = (n + 1) (x1 + x2 + · · · + xn). Ta suy ra S (Π) + SΠ = n + 1 = 2r. Nhưng vì |S (Π0)| > r và SΠ > r nên ta phải có S (Π0) và SΠ trái dấu nhau, nghĩa là một số thì lớn hơn r, còn một số thì nhỏ hơn −r. Từ Π0, ta có thể thu được Πm = Π bằng cách chuyển hai phần tử kề nhau. Nói cách khác, tồn tại một dãy các hoán vị Π0, Π1, . . . , Πm sao cho Πm = Π và với mỗi i(i = 0, 1, . . . , m − 1), hoán vị Πi+1 có được từ Πi bằng cách hoán chuyển hai số hạng liên tiếp. Điều này có nghĩa rằng nếu Πi = (y1, y2, . . . , yn), Πi+1 = (z1, z2, . . . , zn) thì tồn tại một chỉ số k ∈ {1, 2, . . . , n − 1} sao cho zk = yk+1, zk+1 = yk, zj = yj, j 6= k, k + 1. Do giả thiết |xi| 6 r, ∀i = 1, 2, . . . , n nên ta có |S (Πi+1) − S (Πi)| = |kzk + (k + 1) zk+1 − kyk − (k + 1) yk+1| = |yk − yk+1| 6 2r. Điều này chứng tỏ rằng khoảng cách giữa hai số liên tiếp bất kỳ trong dãy số S (Π0), . . . , S (Πm) không lớn hơn 2r. Mặt khác, có thể xem S (Π0), S (Πm) là những điểm nằm trên đường thẳng thực, nằm ngoài đoạn [−r; r], từ đó, suy ra rằng ít nhất một trong các số S (Πi) phải rơi vào đoạn đó; nói cách khác; tồn tại Πi để S (Πi) 6 r. Trên đây chúng tôi đã nói về khái niệm tập hợp trù mật cũng như một số ứng dụng của nó trong giải toán sơ cấp nhằm phục vụ cho các kỳ thi Olympic toán phổ thông. Việc tiếp tục nghiên cứu vấn đề này sẽ được nói trong trương trình toán ở bậc đại học và các bậc học tiếp theo. 3. Bài tập tự luyện Bài tập 1. Cho X, Y là hai tập con của R sao cho X trù mật trong R và X ⊂ Y. Chứng minh rằng Y trù mật trong R. Bài tập 2. Cho X ⊂ R và X trù mật trong R, và A là tập con hữu hạn của X. Chứng minh rằng X\A trù mật trong R. 66 Tạp chí Epsilon, Số 09, 06/2016 Bài tập 3. Chứng minh rằng tồn tại vô số (m, n) ∈ Z2sao cho n√2 = 2m. Bài tập 4. (Isarel – Hungarian 1998) Tìm tất cả α ∈ R thoả mãn ∀n ∈ N∗, ∃m ∈ Z : α −mn <13n. Bài tập 5. Cho a > b là hai số nguyên dương thoả mãn ab > 4. Chứng minh rằng tồn tại số nguyên dương l sao cho b < ϕ 2l = 2l−1 < 2l < a. Bài tập 6. Cho tập hợp S = n√a −√b | a, b ∈ No. Chứng minh rằng tập S trù mật trong R. Bài tập 7. Cho các dãy số (an)n>0và (bm)m>0thoả mãn đồng thời các điều kiện sau: lim n→+∞an = lim m→+∞bm = +∞, và lim m→+∞(bm+1 − bm) = 0. Chứng minh rằng tập S = {an − bm | m, n ∈ N∗} là trù mật trong R. Bài tập 8. Cho (an) và (bn) là hai dãy số dương, tăng ngặt và không bị chặn. Đặc biệt, tập hợp {|an+1 − an| | n ∈ N∗} bị chặn. n Chứng minh rằng tập hợp S = bn| m, n ∈ N∗ am o trù mật trong R+. Bài tập 9. Cho hàm f : R+ → R+ thoả mãn các điều kiện sau: (i) f khả vi trên R+. (ii) lim x→+∞f (x) = +∞. (iii) |f0(x)| < M, ∀x ∈ R+\ (0; 1). Cho (bn) là dãy số dương, tăng và không bị chặn. Chứng minh khi đó tập S = nf(m) bn| m, n ∈ N∗ o trù mật trong R+. Do vậy tập n √n 2m | m, n ∈ N∗ o trù mật trong R+. Bài tập 10. Cho a, b ∈ Z+ sao cho s (an) = s (bn), ∀n ∈ Z+. Chứng minh rằng log ab∈ Z. Trong đó, s (x) là tổng các chữ số của x viết trong hệ nhị phân. Tài liệu tham khảo [1] Titu Andreescu, Gabriel Dospinescu, Problems from the book. [2] Art of Problem Solving: http://artofproblemsolving.com 67 Tạp chí Epsilon, Số 09, 06/2016 68 ABOUT MEANS OF NON-NEGATIVE NUMBERS AND POSITIVE DEFINITE MATRICES Đinh Trung Hòa (Department of Fundamental Sciences, Ho Chi Minh University of Food Industry) Trong bài viết ngắn này, tác giả Đinh Trung Hòa sẽ giới thiệu về lý thuyết trung bình của các số không âm và các ma trận xác định dương. Khởi đầu từ các khái niệm quen thuộc như trung bình cộng, trung bình nhân, trung bình điều hòa của hai số, tác giả đi đến khái quát hóa khái niệm trung bình. Tiếp theo bài viết giới thiệu các tính chất đặc trưng của hàm đơn điệu liên quan đến các trung bình. Cuối cùng, tác giả giới thiệu định nghĩa các dạng trung bình của hai ma trận. Từ các định nghĩa này và các bất đẳng thức số và hàm, các vấn đề tương ứng với ma trận sẽ được đặt ra một cách tự nhiên (và tác giả cùng với một số tác giả khác cũng đã thu được một số kết quả theo hướng này). Bài viết này dựa trên các bài nói chuyện của TS Đinh Trung Hòa tại Đà Nẵng (cho nhóm học sinh THPT chuyên Lê Quý Đôn Đà Nẵng chuẩn bị dự thi HSG quốc gia năm 2014) và trong buổi seminar dành cho học sinh và sinh viên tại trung tâm Titan Education (năm 2015). Đây là bài viết ngắn, văn phong dễ đọc nên chúng tôi để nguyên bản tiếng Anh. Để độc giả tiện theo dõi, chúng tôi xin chuyển ngữ và giải thích một số thuật ngữ tiếng Anh. Positive definite matrices = ma trận xác định dương Positive semidefinite = nửa xác định dương Successive iterations = các bước lặp liên tiếp Arithmetic, geometric, harmonic means = trung bình cộng, trung bình nhân, trung bình điều hòa. Two-variable function = hàm 2 biến Monotone increasing = đơn điệu tăng One-to-one map = ánh xạ 1 − 1 Reverse Cauchy inequality = bất đẳng thức Cauchy ngược Convex function = hàm lồi Classification = phân loại Quantum information theory = Lý thuyết thông tin lượng tử Matrix optimization = Tối ưu hóa ma trận Hermitian matrices = ma trận Hermite 69 Tạp chí Epsilon, Số 09, 06/2016 Eigenvalues = giá trị riêng Eigenspaces = không gian riêng Orthogonal projections = hình chiếu vuông góc Axiomatic approach = cách tiếp cận tiên đề Binary operation = phép toán 2 ngôi Riemannian manifold = đa tạp Riemann Weighted Heron mean = trung bình Heron có trọng Norht and south poles = bắc cực và nam cực Mathematical aspects = khía cạnh toán học Theory of operator and matrix = lý thuyết toán tử và ma trận In this short we introduce the theory of means for non-negative numbers and positive definite matrices. 1. Some information about means for scalars 1.1. Some well-known means We list some (not all) definitions of the geometric mean as follows: • The unique solution of xa−1x = b; • The common list of the successive iterations of harmonic and arithmetic means: √ab = lim an = lim bn, where a0 = a, b0 = b, an+1 =2anbn an+bn, bn+1 =an+bn 2. • The maximum among real x for which nant is nonnegative: x2 6 ab. a x x b is positive semidefinite, or its determi • The unique midpoint for the metric δ on R+ given by δ(x, y) = δ(a, b) = 2δ(a, √ab) = 2δ(b, √ab); log xy : • If x(t) is the solution of x0 = b − a−1x with initial condition x(0) = x0 > 0, then limt→∞ x(t) = √ab. It is well-known that for two nonnegative numbers a, b and s ∈ [0, 1] 2)−1 6√ab 6asb1−s + a1−sbs (a−1 + b−1 2)−1is the harmonic mean, asb1−s+a1−sbs 26a + b 2, (1.1) where (a−1+b−1 2is the Heinz mean and a+b 2is the arith metic mean. 70 Tạp chí Epsilon, Số 09, 06/2016 The AM-GM inequality states that only the square has the smallest perimeter amongst all rectangles of equal area. At the same time, we have the following geometric interpretation: The geometric mean and the arithmetic mean are the midpoints of curves asb1−sand sa + (1 − s)b, respectively, connecting a and b. And the Heinz mean is the curve connecting the geometric mean and the arithmetic mean when s runs from 0 to 1. Now, let us formulate a general approach of means. A mean M of nonnegative numbers is a map from R+ × R+ to R+ such that: 1) M(x, x) = x for every x ∈ R+; 2) M(x, y) = M(y, x) for every x, y ∈ R+; 3) If x < y, then x < M(x, y) < y; 4) If x < x0 and y < y0, then M(x, y) < M(x0, y0); 5) M(x, y) is continuous; 6) M(tx, ty) = tM(x, y) (t, x, y ∈ R+. A two-variable function M(x, y) satisfying condition (6) can be reduced to a one-variable function f(x) := M(1, x). (1.2) Namely, M(x, y) is recovered from f as M(x, y) = xf(x−1y). Mention that the corresponding to M function f is monotone increasing on R+. And that relation (1.2) forms a one-to-one map between means and monotone increasing functions on R+. It is obvious that: • The arithmetic mean is corresponding to 1+t 2; • The geometric mean is corresponding to √t; • The harmonic mean is corresponding to 2t 1+t; • The log-mean L(x, y) = x−y log x−log yis corresponding to t−1 log t. 1.2. Monotone functions and means Let f be a monotone increasing function on [0,∞), then for any nonnegative number a, b, f(√ab) 6 f a + b 2 . (1.3) It is natural to ask that if inequality (1.3) holds for any pair of positive number a, b will the function f be monotone increasing on [0,∞)? The answer is positive, and follows from the elementary fact that for any positive numbers a 6 b there exist positive number x, y such that a is arithmetic mean and b is geometric mean of x, y. An interesting and useful reverse Cauchy inequality is as follows: For any positive number x, y, x + y 26√xy +12|x − y|, (1.4) or min{x, y} 6√xy. 71 Tạp chí Epsilon, Số 09, 06/2016 Proposition 1. A function f on [0,∞) is monotone increasing if and only if the following inequality f(x + y 2) 6 f hold for any pair of positive number x, y. √xy +12|x − y| , (1.5) Chứng minh. Firstly, we show that for 0 6 a 6 b 6√2a there exist positive number x, y such that a =x + y 2, b =√xy +12|x − y|. We can assume that x 6 a. It is easy to see that for 0 6 a 6 b 6√2a the equation b =px(2a − x) + a − x, or, 2x2 + 2(b − 2a)x + (b − a)2 = 0 has a positive solution. Consequently, if we have (1.5), then f(a) = x + y 26 f √xy +12|x − y| = f(b). For arbitrary a 6 b, it is obvious that there exist numbers a1, a2, . . . , am such that a = a0 6 a1 6 · · · 6 am = b and ai 6 ai+1 6√2ai. Apply above arguments, we can get f(a) 6 f(a1) 6 f(a2) 6 · · · 6 f(an) = f(b). Of course we can get new characterization of monotone increasing functions by using other inequalities in (1.1). 1.3. Some classes of convex functions with means Let M, N be arbitrary means and f be continuous on R+. We call f to be MN-convex if for any nonnegative x, y, f(M(x, y)) 6 N(f(x), f(y)). (1.6) If M(x, y) = N(x, y) = sx + (1 − s)y, then we have the class of convex functions, and inequality (1.6) is no thing but Jensen inequality for convex functions. Such functions have many applications in optimization, in information theory etc. If M(x, y) = sx + (1 − s)y, N(x, y) = xsy1−s, then we have the class of log-convex functions. If M(x, y) = N(x, y) = xsy1−s, then we have the class of geometrically convex functions. According to applications of several kinds of convexity many mathematicians give effort to study the problem of classification of convex functions. And a such problem is more complicated for matrices. 72 Tạp chí Epsilon, Số 09, 06/2016 2. A little about matrix means In this section we talk about the matrix means that have many applications in quantum information theory, matrix optimizations etc. Let Mn be the space of n×n complex matrices. For A, B ∈ Mn, the notation A 6 B means that B − A ∈ M+n(i.e., < (B − A)x, x > > 0 for any vector x). The spectrum of a matrix A ∈ Mn is denoted by σ(A). For a real-valued function f of a real variable and a matrix A ∈ Mhn, the value f(A) is understood by means of the functional calculus for Hermitian matrices, i.e. if A =PλiPi (where λi are eigenvalues of A, Pi are orthogonal projections on the eigenspaces of λi), then the matrix f(A) is defined as Pf(λi)Pi. We can understand f(A) in another way: Let A = U∗diag(λ1, λ2, . . . , λn)U (where λi are the eigenvalues of A), then f(A) = U∗diag(f(λ1), . . . , f(λn))U. Taking an axiomatic approach, Kubo and Ando [6] introduced the notions of connection and mean. A binary operation σ defined on the set of positive definite matrices is called a connection if (i) A 6 C, B 6 D imply AσB 6 BσD; (ii) C∗(AσB)C 6 (C∗AC)σ(C∗BC); (iii) An ↓ A and Bn ↓ B imply AnσBn ↓ AσB If IσI = I, then σ is called a mean. For A, B > 0, the t-geometric mean of positive definite matrices A, B is A]tB := A1/2A−1/2BA−1/2 tA1/2. (2.1) When t = 1/2, A]B := A]1/2B is called the geometric mean of A and B, and it was first introduced in [8] and is often denoted by A]B in the literature (the problem of defining the geometric mean of two positive definite matrices was standing for more than 25 years because we could not just take the square root of product of two positive definite matrices AB!!). The harmonic A!B and arithmetic A∇B means are defined A!B = 2(A−1 + B−1)−1and A∇B =A + B 2, respectively. A mean σ is called to be symmetric if AσB = BσA for any pair of positive definite matrices A, B. It is well-known that the arithmetic mean ∇ is the biggest among symmetric means. From the general theory of matrix means we know that ∇ > σ and τ >!. And we still have the following inequalities A!B 6 A]B 6 A∇B. (2.2) The proof of (2.2) is not very difficult and based on the scalar case. The t-geometric mean is interesting from the point of view of Riemannian geometry since the set Pn of positive semidefinite matrices is a Riemannian manifold with the Riemannian metric [9]: δ(A, B) = Xn i=1 log2λi(A−1B) !1/2 , A, B ∈ Pn, which is the distance between A and B so that the curve β(t) = A]tB, 0 6 t 6 1, is the unique geodesic joining A and B in Pn. The picture is more interesting if we consider the weighted Heron mean (1 − λ)A+B 2 + λA]B(λ ∈ [0, 1]) that connects two midpoints A+B 2and A]B (this model looks like the earth with A and B are the north and south poles, respectively!). 73 Tạp chí Epsilon, Số 09, 06/2016 3. Concluding comment Many properties formulated in Subsection 1.1 have been studied for matrices [10]. Some in teresting information related to the last part of Subsection 1.1 an be found in [1] (Petz is one of the most active mathematicians in studying mathematical aspects of quantum information theory) where the author discusses about means of more than 2 positive semidefinite matrices. Matrix analogs of results in Subsection 1.2 recently was studied by Ando and Hiai [11] and by the author [12]. At that time, problems of characterization of matrix convex functions in sense of Subsection 1.3 are still in research interests of many mathematicians. Nowadays, the theory of operator and matrix means is one of the most studied in matrix analysis with many applications in machine learning and quantum theory etc. Tài liệu tham khảo [1] D.Petz. Means of positive matrices: Geometry and a conjecture. Annales Mathematicae et Informaticae. 32 (2005) 129-139. [2] F. Topse. Basic Concepts, Identities and Inequalities theToolkit of Information Theory. Entropy, 3 (2001) 162. [3] K. M. R.Audenaert, J. Calsamiglia, LI. Masanes, R. Munoz-Tapia, A. Acin, E. Bagan and F. Verstraete, Discriminating States: The Quantum Chernoff Bound, Phys. Rev. Lett. 98 (2007) 160501. [4] D. Bini, B. Meini, F. Poloni, An effective matrix geometric mean satisfying the Ando-Li Mathias properties, Math. Comp. 79 (2010) 437-452. [5] S. Furuichi, K. Yanagi, and K. Kuriyama, Fundamental properties of Tsallis relative entropy, J. Math. Phys. 45(12) (2004) 4868-4877. [6] F. Kubo and T. Ando, Means of positive linear operators, Math. Ann. 246(3) (1980) 205-224. [7] H. Lee, Y. Lim, T. Yamazaki, Multi-variable weighted geometric means of positive definite matrices, Linear Algebra Appl. 435(2) (2011) 307-322. [8] W. Pusz, S.L. Woronowicz, Functional calculus for sesquilinear forms and the purification map, Rep. Math. Phys., 8 (1975), 159-170. [9] R. Bhatia, Positive Definite Matrices, Princeton University Press, New Jersey, 2007. [10] J. D. Lawson, Y. Lim, The geometric mean, matrices, metrics, and more, Amer. Math. Monthly 108 (2001), 797-812. [11] T. Ando, F. Hiai, Log majorization and complementary Golden-Thompson type inequalities, Linear Algebra Appl., 197/198 (1994), 113–131. [12] Dinh Trung Hoa. On characterization of operator monotone functions. Linear Algebra and Its Applications. 487 (2015) 260-267. 74 ĐIỂM KOSNITA VÀ MỘT SỐ ĐƯỜNG THẲNG ĐI QUA NÓ Trịnh Huy Vũ (Lớp 12 A1 Toán, trường THPT Chuyên KHTN, Hà Nội) Bài viết nêu ra định nghĩa và một số kết quả về điểm Kosnita. Đồng thời bài báo cũng sẽ đưa ra chứng minh thuần túy cho một số đường thẳng đi qua điểm Kosnita. 1. Định nghĩa Định nghĩa 2. Điểm Kosnita K của 4ABC bất kì được xác định là điểm đồng quy của ba đường thẳng nối ba đỉnh A, B, C tương ứng với tâm ngoại tiếp của 4OBC, 4OCA, 4OAB với O là tâm ngoại tiếp của 4ABC. Điểm Kosnita là Kimberling center X(54) trong Encyclopedia of Triangle Center (ETC) [1]. A E F O K D B C Hình 11.1: Điểm Kosnita 2. Một số kết quả quen thuộc Sau đây, tác giả sẽ trình bày ra một số kết quả quen thuộc về điểm Kosnita mà tác giả sẽ dùng trong bài viết. 75 Tạp chí Epsilon, Số 09, 06/2016 Kết quả 1. 4ABC có tâm đường tròn Euler N và điểm Kosnita K. Khi đó, N và K đẳng giác trong 4ABC. Định nghĩa 3. Cho tam giác ABC với tâm nội tiếp I. Đường thẳng Euler của 4IBC, 4ICA, 4IAB, 4ABC đồng quy tại một điểm. Điểm đồng quy đó được gọi là điểm Schiffler của 4ABC. Kết quả 2. Gọi D, E, F lần lượt là trung điểm của các cung nhỏ BC, CA, AB của 4ABC. Khi đó, điểm Kosnita của 4DEF cũng là điểm Schiffler của 4ABC. Chứng minh. X là trung điểm BC. Vẽ đường kính DJ. Lấy các điểm D0, E0, F0lần lượt đối xứng D, E, F qua BC, CA, AB. Gọi N, U, V, W tương ứng là trung điểm của IO, ID0, IE0, IF0. Ta có các kết quả quen thuộc: D, E, F tương ứng là tâm ngoại tiếp của các tam giác IBC , ICA , IAB; I, N là trực tâm và tâm đường tròn Euler của 4DEF. Ta có DI2 = DB2 = DC2 = DX.DJ (theo hệ thức lượng trong tam giác vuông). Từ đó ta được DI2 = DX.DJ = 2DX.DO = DD0.DO. Suy raDO DI=DI DD0. Do đó, 4DOI ∼ 4DID0(c.g.c). Mà hai tam giác này có hai trung tuyến tương ứng là DN và DU, do vậy ∠NDI = ∠UDO. Nói cách khác, DN và DU đẳng giác trong ∠IDO. Mặt khác, do I, O tương úng là trực tâm và tâm ngoại tiếp của 4DEF nên DH, DO đẳng giác trong ∠EDF. Từ đó ta thu được DN và DU đẳng giác trong ∠EDF. Mặt khác ta lại có U là tâm đường tròn Euler của 4IBC nên DU chính là đường thẳng Euler của 4IBC. Vậy DN và đường thẳng Euler của 4IBC đẳng giác trong 4DEF. Chứng minh tương tự ta cũng được EN, đường thẳng Euler của 4ICA và F N, đường thẳng Euler của 4IAB là hai cặp đường thẳng đẳng giác trong 4DEF. Do vậy điểm Schiffler S đẳng giác với N qua 4DEF. Theo Kết quả 1, điểm Schiffler S của 4ABC cũng là điểm Kosnita của 4DEF J E A O F I U N D0 X BC D Hình 11.2: Điểm Kosnita của 4DEF và điểm Schiffler của 4ABC trùng nhau 76 Tạp chí Epsilon, Số 09, 06/2016 Nhận xét. Kết quả 2 còn có cách chứng minh khác của Telv Cohl trong [4]. Từ kết quả này thu được hệ quả là điểm Kosnita của 4DEF nằm trên đường thẳng Euler của 4ABC. Hơn nữa, dựa sự trùng nhau của điểm Schiffler và điểm Kosnita của 4ABC và 4DEF, ta cũng thu được chứng minh của kết quả sau: Kết quả 3. Gọi D, E, F lần lượt là trung điểm của các cung nhỏ BC, CA, AB của đường tròn (O) ngoại tiếp 4ABC. H là trực tâm của 4ABC và K là điểm Kosnita của 4DEF. Khi đó, OK OH =R ABC. 2r + 3Rvới R, r tương ứng là bán kính của đường tròn ngoại tiếp và nội tiếp tam giác G Ha E A L F I H K O X BC D Hình 11.3: Tỉ số OK/OH Trước hết ta nêu ra một bổ đề quen thuộc, tác giả xin phép không nêu chứng minh của bổ đề này. Bổ đề 5. Cho 4ABC với trực tâm H, tâm ngoại tiếp O và M là trung điểm BC. Khi đo, AH = 2OM. Chứng minh. L là hình chiếu của I lên AC. X là trung điểm BC. Gọi Ha là trực tâm của 4IBC. Do K nằm trên đường thẳng Euler của 4IBC nên K ∈ DHa. Đặt DHa cắt AH tại G. Do 4ALI ∼ 4BXD(g.g) nênDI AI =DB AI =XD IL=XD suy raIHa AG =DI XD + r. Theo bổ đề thì 2DX r. Từ đó áp dụng định lý Thales, ta DA =XD AG =XD XD + r. Do vậy, AG = 2(XD + r), 77 Tạp chí Epsilon, Số 09, 06/2016 suy ra HG = AH + AG = 2OX + 2(XD + r) = 2(r + R). Theo định lý Thales suy ra KH=OD HG =R 2(r + R)hay OK OK OH =R 2r + 3R. 3. Một số đường thẳng đi qua điểm Kosnita Trong mục này tác giả sẽ phát biểu một số đường thẳng đi qua điểm Kosnita. Hầu hết các đường thẳng này đều đã được liệt kê trong ETC [1] nên tác giả chỉ phát biểu lại và đồng thời đưa ra những chứng minh thuần túy hình học cho chúng. 3.1. Đường thẳng đi qua ảnh nghịch đảo của trực tâm H qua đường tròn ngoại tiếp (O) và tâm của đường tròn Taylor (X(186)X(389)) Định nghĩa 4. 4ABC với A0, B0, C0tương ứng là chân ba đường cao. AB, AC lần lượt là hình chiếu của A0trên AB, AC. Xác đinh các điểm BC, BA, CA, CB tương tự. Khi đó, 6 điểm AB, AC, BC, BA, CA, CB cùng nằm trên một đường tròn. Đường tròn này được gọi là đường tròn Taylor [5]. Tâm đường tròn Taylor trong ETC là Kimberling center X(389) [1]. A BA CA B0 C0 AB AC B A0 C CB BC Hình 11.4: Đường tròn Taylor Chứng minh đường tròn Taylor. Ta thực hiện phép biến đổi góc sau: ∠AABAC = ∠AA0AC (A, AB, A0, AC nằm trên đường tròn (AA0)) = ∠ACB (cùng phụ ∠A0AC) = ∠AC0B0(B, C, B0, C0nằm trên đường tròn (BC)) = ∠ACABA (B0, C0, BA, CA nằm trên đường tròn(B0C0)) Do vậy 4 điểm AB, AC, BA, CA cùng thuộc một đường tròn. Chứng minh tương tự thì BABCCBAB và CACBBCAC đều là các tứ giác nội tiếp đường tròn. Giả sử ba đường tròn này không trùng nhau thì ta thấy BCCB, CAAC, ABBA lần lượt là các trục đẳng phương của 2 trong bộ 3 đường tròn {(ABACBACA); (BABCCBAB); (CACBBCAC)}, tức là BCCB, CAAC, ABBA phải đồng quy (mâu thuẫn). Do vậy, 3 đường tròn này trùng nhau và 6 điểm AB, AC, BC, BA, CA, CB cùng thuộc 1 đường tròn. 78 Tạp chí Epsilon, Số 09, 06/2016 Các chứng minh khác cho đường tròn Taylor tham khảo thêm trong [6], [7]. Định nghĩa 5. Cho 4ABC với D, E, F lần lượt là trung điểm của BC, CA, AB. Khi đó, tâm nội tiếp của 4DEF được gọi là điểm Spieker của 4ABC. Sau đây tác giả sẽ trình bày thêm một tính chất liên quan đến tâm đường tròn Taylor X(389) và điểm Spieker. Bổ đề 6. 4ABC nhọn với A0, B0, C0lần lượt là ba chân đường cao. Khi đó, tâm của đường tròn Taylor của 4ABC là điểm Spieker của 4A0B0C0. A BA CA B0 D C0 AB E S F AC B A0 C CB BC Hình 11.5 Chứng minh. S là tâm đường tròn Taylor của 4ABC. Gọi D, E, F lần lượt là trung điểm của B0C0, C0A0, A0B0. Trước hết ta chứng minh ABAC đi qua E, F. Thật vậy, do E là trung điểm của C0A0 nên ∠EABA0 = ∠C0A0AB = 90◦ −∠BC0A0 = 90◦ −∠ACB = ∠A0AAC = ∠ACABA0. Do đó, E ∈ ABAC. Tương tự với F suy ra ABAC đi qua E, F. Chứng minh tương tự như trên ta được BABC đi qua F, D và CACB đi qua D, E. Mặt khác dễ thấy BACA k CBBC nên BACABCCB là một hình thang cân, mà D = BABC ∩ CACB suy ra 4DBCCB là tam giác cân và DS là phân giác của ∠BCDCB ≡ ∠EDF. Tương tự, ES là phân giác của ∠DEF. Vậy S là tâm nội tiếp của 4DEF và cũng là điểm Spieker của 4A0B0C0. Nhận xét. Kết quả trên chỉ đúng khi 4ABC là tam giác nhọn. Khi 4ABC tù lần lượt tại A, B, C thì tâm đường tròn Taylor của 4ABC sẽ là tâm bàng tiếp tương ứng với các đỉnh D, E, F của tam giác trung bình DEF của 4A0B0C0. Tác giả xin tiếp tục nêu thêm ra một kết quả nổi tiếng liên quan đến điểm Spieker: Bổ đề 7. 4ABC với tâm nội tiếp I, trọng tâm G và điểm Spieker Sp và điểm Nagel Na. Khi đó, I, G, Sp, Na cùng nằm trên một đường thẳng và INa = 2ISp = 3IG. Đây là một định lý nổi tiếng, bạn đọc có thể tham khảo chứng minh của nó từ trang 7-12 trong [6]. Định lý 1. Cho 4ABC với trực tâm H, đường tròn ngoại tiếp (O) và 4DEF là tam giác tạo bởi ba chân đường cao của 4ABC. S là tâm đường tròn Taylor của 4ABC. H∗là ảnh nghịch đảo của H qua (O). Khi đó, đường thẳng SH∗ đi qua điểm Kosnita K của 4ABC. 79 Tạp chí Epsilon, Số 09, 06/2016 A E J X Ho S Z K R O F H Y G P Q N H∗ B LM C D Hình 11.6: Định lý 3.1 Trong đó, ảnh nghịch đảo H∗của H của (O) là Kimberling center X(186) trong ETC [1]. Trong bài viết này tác giả sẽ nêu chứng minh của định lý 1 trong trường hợp 4ABC nhọn (các trường hợp còn lại chứng minh tương tự). Khi đó, tâm đường tròn Taylor chính là điểm Spieker của tam giác DEF. Do vậy, ta quy bài toán về việc chứng minh đường thẳng đi qua điểm Spieker của 4DEF và ảnh nghịch đảo của H qua (O) đi qua điểm Kosnita K. Chứng minh định lý. N là tâm đường tròn Euler của 4ABC và P là trung điểm của NO. X, Y, Z, J, L, M tương ứng là trung điểm của EF, F D, DE, HA, HB, HC. Dễ thấy H là tâm nội tiếp của 4DEF (kết quả nổi tiếng). Gọi Ho, G lần lượt là trực tâm và trọng tâm của 4DEF. Ta được Ho, G, N thẳng hàng trên đường thẳng Euler của 4DEF và H, G, S thẳng hàng, HG = 2GS. Gọi Q là trung điểm của HK. r, R lần lượt là bán kính đường tròn nội tiếp và ngoại tiếp của 4DEF. Ta có OA2 = OH.OH∗suy raOH OH∗= AH AH∗ 2 = OH OA 2 =NH2 R2. Mặt khác, áp dụng công thức Euler và chú ý là H, N lần lượt là tâm nội tiếp và tâm ngoại tiếp của 4DEF, thì NH2 = Rr2 − 2Rr = R(R − 2r). Do đó, OH R. VậyOP P H∗ OH∗=3R + 2r 4R. OH∗=R − 2r OH∗=R − 2r 4Rhay Xét phép vị tự tâm H tỉ số 12, A 7→ J; B 7→ L; C 7→ M; K 7→ Q, do đó Q là điểm Kosnita của 4JLM. Mặt khác, ta có kết quả nổi tiếng J, L, M lần lượt là trung điểm các cung EF, F D, DE của đường tròn (DEF). Theo kết quả 2.2, 2.3 thì Q là điểm Schiffler của 4DEF, Q ∈ HoN và 80